Емкость конденсатора на что влияет: §52. Конденсаторы, их назначение и устройство

Содержание

§52. Конденсаторы, их назначение и устройство

Заряд и разряд конденсатора.

Конденсатор представляет собой устройство, способное накапливать электрические заряды. Простейшим конденсатором являются две металлические пластины (электроды), разделенные каким-либо диэлектриком. Конденсатор 2 можно зарядить, если соединить его электроды с источником 1 электрической энергии постоянного тока (рис. 181, а).

Рис. 181. Заряд и разряд конденсатора

При заряде конденсатора свободные электроны, имеющиеся на одном из его электродов, устремляются к положительному полюсу источника, вследствие чего этот электрод становится положительно заряженным. Электроны с отрицательного полюса источника устремляются ко второму электроду и создают на нем избыток электронов, поэтому он становится отрицательно заряженным.

В результате протекания зарядного тока i3 на обоих электродах конденсатора образуются равные, но противоположные по знаку заряды и между ними возникает электрическое поле, создающее между электродами конденсатора определенную разность потенциалов. Когда эта разность потенциалов станет равной напряжению источника тока, движение электронов в цепи конденсатора, т. е. прохождение по ней тока i3 прекращается. Этот момент соответствует окончанию процесса заряда конденсатора.

При отключении от источника (рис. 181,б) конденсатор способен длительное время сохранять накопленные электрические заряды. Заряженный конденсатор является источником электрической энергии, имеющим некоторую э. д. с. ес. Если соединить электроды заряженного конденсатора каким-либо проводником (рис. 181, в), то конденсатор начнет разряжаться.

При этом по цепи пойдет ток iр разряда конденсатора. Начнет уменьшаться и разность потенциалов между электродами, т. е. конденсатор будет отдавать накопленную электрическую энергию во внешнюю цепь.

В тот момент, когда количество свободных электронов на каждом электроде конденсатора станет одинаковым, электрическое поле между электродами исчезнет и ток станет равным нулю. Это означает, что произошел полный разряд конденсатора, т. е. он отдал накопленную им электрическую энергию.

Емкость конденсатора.

Свойство конденсатора накапливать и удерживать электрические заряды характеризуется его емкостью. Чем больше емкость конденсатора, тем больше накопленный им заряд, так же как с увеличением вместимости сосуда или газового баллона увеличивается объем жидкости или газа в нем.

Емкость С конденсатора определяется как отношение заряда q, накопленного в конденсаторе, к разности потенциалов между его электродами (приложенному напряжению)U:

C = q / U (69)

Емкость конденсатора измеряется в фарадах (Ф). Емкостью в 1 Ф обладает конденсатор, у которого при сообщении заряда в 1 Кл разность потенциалов возрастает на 1 В. В практике преимущественно пользуются более мелкими единицами: микрофарадой (1 мкФ=10-6 Ф), пикофарадой (1 пФ = 10

-12 мкФ).

Емкость конденсатора зависит от формы и размеров его электродов, их взаимного расположения и свойств диэлектрика, разделяющего электроды. Различают плоские конденсаторы, электродами которых служат плоские параллельные пластины (рис. 182, а), и цилиндрические (рис. 182,б).

Рис. 182. Плоский (а) и цилиндрический (б) конденсаторы

Свойствами конденсатора обладают не только специально изготовленные на заводе устройства, но и любые два проводника, разделенные диэлектриком. Емкость их оказывает существенное влияние на работу электротехнических установок при переменном токе.

Например, конденсаторами с определенной емкостью являются два электрических провода, провод и земля (рис. 183, а), жилы электрического кабеля, жилы и металлическая оболочка кабеля (рис. 183,6).

Рис. 183. Емкости, образованные проводами воздушной линии (а) и жилами кабеля (б)

Устройство конденсаторов и их применение в технике.

В зависимости от применяемого диэлектрика конденсаторы бывают бумажными, слюдяными, воздушными (рис. 184).

Рис. 184. Общие виды применяемых конденсаторов: 1 — слюдяные; 2 — бумажные; 3 — электролитический; 4 — керамический

Используя в качестве диэлектрика вместо воздуха слюду, бумагу, керамику и другие материалы с высокой диэлектрической проницаемостью, удается при тех же размерах конденсатора увеличить в несколько раз его емкость. Для того чтобы увеличить площади электродов конденсатора, его делают обычно многослойным.

В электротехнических установках переменного тока обычно применяют силовые конденсаторы. В них электродами служат длинные полосы из алюминиевой, свинцовой или медной фольги, разделенные несколькими слоями специальной (конденсаторной) бумаги, пропитанной нефтяными маслами или синтетическими пропитывающими жидкостями.

Ленты фольги 2 и бумаги 1 сматывают в рулоны (рис. 185), сушат, пропитывают парафином и помещают в виде одной или нескольких секций в металлический или картонный корпус. Необходимое рабочее напряжение конденсатора обеспечивается последовательным, параллельным или последовательно-параллельным соединениями отдельных секций.

Рис. 185. Устройство бумажного (а) и электролитического (б) конденсаторов

Всякий конденсатор характеризуется не только значением емкости, но и значением напряжения, которое выдерживает его диэлектрик. При слишком больших напряжениях электроны диэлектрика отрываются от атомов, диэлектрик начинает проводить ток и металлические электроды конденсатора замыкаются накоротко (конденсатор пробивается).

Напряжение, при котором это происходит, называют пробивным. Напряжение, при котором конденсатор может надежно работать неограниченно долгое время, называют рабочим. Оно в несколько раз меньше пробивного.

Конденсаторы широко применяют в системах энергоснабжения промышленных предприятий и электрифицированных железных дорог для улучшения использования электрической энергии при переменном токе.

На э. п. с. и тепловозах конденсаторы используют для сглаживания пульсирующего тока, получаемого от выпрямителей и импульсных прерывателей, борьбы с искрением контактов электрических аппаратов и с радиопомехами, в системах управления полупроводниковыми преобразователями, а также для создания симметричного трехфазного напряжения, требуемого для питания электродвигателей вспомогательных машин.

В радиотехнике конденсаторы служат для создания высокочастотных электромагнитных колебаний, разделения электрических цепей постоянного и переменного тока и др.

В цепях постоянного тока часто устанавливают электролитические конденсаторы. Их изготовляют из двух скатанных в рулон тонких алюминиевых лент 3 и 5 (рис. 185,б), между которыми проложена бумага 4, пропитанная специальным электролитом (раствор борной кислоты с аммиаком в глицерине).

Алюминиевую ленту 3 покрывают тонкой пленкой окиси алюминия; эта пленка образует диэлектрик, обладающий высокой диэлектрической проницаемостью. Электродами конденсатора служат лента 3, покрытая окисной пленкой, и электролит; вторая лента 5 предназначена лишь для создания электрического контакта с электролитом. Конденсатор помещают в цилиндрический алюминиевый корпус.

При включении электролитического конденсатора в цепь постоянного тока необходимо строго соблюдать полярность его полюсов; электрод, покрытый окисной пленкой, должен быть соединен с положительным полюсом источника тока. При неправильном включении диэлектрик пробивается.

По этой причине электролитические конденсаторы нельзя включать в цепи переменного тока. Их нельзя также использовать в устройствах, работающих при высоких напряжениях, так как окисная пленка имеет сравнительно небольшую электрическую прочность.

В радиотехнических устройствах применяют также конденсаторы переменной емкости (рис. 186).

Рис. 186. Устройство конденсатора переменной емкости

Такой конденсатор состоит из двух групп пластин: неподвижных 2 и подвижных 3, разделенных воздушными промежутками. Подвижные пластины могут перемещаться относительно неподвижных; при повороте оси 1 конденсатора изменяется площадь взаимного перекрытия пластин, а следовательно, и емкость конденсатора.

Способы соединения конденсаторов.

Конденсаторы можно соединять последовательно и параллельно. При последовательном соединении нескольких (например, трех), конденсаторов (рис. 187, а) эквивалентная емкость

1 /Cэк = 1 /C1 + 1 /C2 + 1 /C3

эквивалентное емкостное сопротивление

XCэк= XC1 + XC2 + XC3

результирующее емкостное сопротивление

Cэк = C1 + C2 + C3

При параллельном соединении конденсаторов (рис. 187,б) их результирующая емкость

1 /XCэк = 1 /XC1 + 1 /XC2 + 1 /XC3

Рис. 187. Последовательное (а) и параллельное (б) соединения конденсаторов

Включение и отключение цепей постоянного тока с конденсатором.

При подключении цепи R-C к источнику постоянного тока и при разряде конденсатора на резистор также возникает переходный процесс с апериодическим изменением тока i и напряжения uc.

При подключении к источнику постоянного тока цепи R-C выключателем В1 (рис. 188,а) происходит заряд конденсатора. В начальный момент зарядный ток Iнач=U /R. Но по мере накопления зарядов на электродах конденсатора напряжение его и с будет возрастать, а ток уменьшаться (рис. 188,б).

Рис. 188. Схема подключения цепи R-C к источнику постоянного тока (а) и кпивые тока и напряжения при переходном процессе (б) кривые

Если сопротивление R мало, то в начальный момент подключения конденсатора возникает большой екачок тока, значительно превышающий номинальный ток данной цепи. При разряде конденсатора на резистор R (размыкается выключатель В1 на рис. 189, а) напряжение на конденсаторе uс и ток i постепенно уменьшаются до нуля (рис. 189,б).

Рис. 189. Схема разряда емкости С на резистор R (а) и кривые тока и напряжения при переходном процессе (б)

Скорость изменения тока i и напряжения ис при переходном процессе отделяется постоянной времени

T = RC

Чем больше R и С, тем медленнее происходит заряд конденсатора.

Процессы заряда и разряда конденсатора широко используют в электронике и автоматике. С помощью их получают периодаческие несинусоидальные колебания, называемые релаксационными, и, в частности, пилообразное напряжение, необходимое для работы систем управления тиристорами, осциллографов и других устройств.

Для получения пилообразного напряжения (рис. 190) периодически подключают конденсатор к источнику питания, а затем к разрядному резистору.

Рис. 190. Кривая пилообразного напряжения

Периоды Т1 и T2, соответствующие заряду и разряду конденсатора, определяются постоянными времени цепей заряда Т3 и разряда Тр, т. е. сопротивлениями резисторов, включенных в эти цепи.

Электрическая емкость. Конденсаторы. Емкость конденсатора.

Электрическая емкость. Конденсаторы.

Емкость уединенного проводника.

Уединенным будем называть проводник, размеры которого много меньше расстояний до окружающих тел. Пусть это будет шар радиусом r. Если потенциал на бесконечности принять за 0, то потенциал заряженного уединенного шара равен:  , где e — диэлектрическая проницаемость окружающей среды.  Следовательно: 

эта величина не зависит ни от заряда, ни от потенциала и определяется только размерами шара (радиусом) и диэлектрической проницаемостью среды. Этот вывод справедлив для проводника любой формы.

 

Электрической емкостью проводника наз. отношение заряда проводника к его потенциалу: .

Емкость определяется геометрической формой, размерами проводника и свойствами среды (от материала проводника не зависит). Чем больше емкость проводника, тем меньше меняется потенциал при изменении заряда.

Емкость шара в СИ:

  —

Единицы емкости.

Емкостью (фарад) обладает такой проводник, у которого потенциал возрастает на 1 В при сообщении ему заряда в 1 Кл.

Емкостью   обладал бы уединенный шар, радиус которого был бы равен 13 радиусам Солнца.

Емкость Земли  700 мкФ

Если проводник не уединенный, то потенциалы складываются по правилу суперпозиции и емкость проводника меняется.

1 мкФ=10-6Ф

1нФ=10-9Ф

1пФ=10-12Ф

Конденсаторы (condensare — сгущение) .

Можно создать систему проводников, емкость которой не зависит от окружающих тел. Первые конденсаторы — лейденская банка (Мушенбрук, сер. XVII в.).

 

Конденсатор представляет собой систему из двух проводников, разделенных слоем диэлектрика, толщина которого мала по сравнению с размерами проводников.  Проводники наз.  обкладками  конденсатора. Если заряды пластин конденсатора одинаковы по модулю и противоположны по знаку, то  под зарядом конденсатора понимают абсолютное значение заряда одной из его обкладок.

На рисунке — плоский и сферический конденсаторы. Поле плоского конденсатора почти все сосредоточено внутри (у идеального — все). Усферического — все поле сосредоточено между обкладками.

 

Электроемкостью конденсатора называют отношение заряда конденсатора к разности потенциалов между обкладками: .

При подключении конденсатора к батарее аккумуляторов происходит поляризация диэлектрика внутри конденсатора и на обкладках появляютсязаряды — конденсатор заряжается. Электрические поля окружающих тел почти не проникают через металлические обкладки и не влияют на разность потенциалов между ними.

 

Емкость плоского конденсатора.

, т.о. емкость плоского конденсатора зависит только от его размеров, формы и диэлектрической проницаемости. Для создания конденсатора большой емкости необходимо увеличить площадь пластин и уменьшить толщину слоя диэлектрика.

Емкость сферического конденсатора .

Если зазор между обкладками мал по сравнению с радиусами, то формула переходит в формулу емкости плоского конденсатора.

Виды конденсаторов

При подключении электролитического конденсатора необходимо соблюдать полярность.

Назначение конденсаторов

  1. Накапливать на короткое время заряд или энергию для быстрого изменения потенциала.
  2. Не пропускать постоянный ток.
  3. В радиотехнике: колебательный контур, выпрямитель.
  4. Фотовспышка.

 

Глава 20. Конденсаторы

Для накопления разноименных электрических зарядов служит устройство, которое называется конденсатором. Конденсатор — система двух изолированных друг от друга проводников (которые часто называют обкладками конденсатора), один из которых заряжен положительным, второй — таким же по величине, но отрицательным зарядом. Если эти проводники представляют собой плоские параллельные пластинки, расположенные на небольшом рас-стоянии друг от друга, то конденсатор называется плоским.

Для характеристики способности конденсатора накапливать заряд вводится понятие электроемкости (часто говорят просто емкости). Емкостью конденсатора называется отношение заряда конденсатора к той разности потенциалов , которая возникает между обкладками при их заряжении зарядами и (эту разность потенциалов проводников часто называют электрическим напряжением между обкладками и обозначают буквой ):

(20.1)

Поскольку величины и (или ) в формуле (20.1) зависимы, то емкость (20.1) не зависит от и , а является характеристикой геометрии системы проводников. Действительно, при сообщении проводникам зарядов и проводники приобретут потенциалы, разность которых будет пропорциональна заряду . Поэтому в отношении (20.1) заряд сокращается.

Выведем формулу для емкости плоского конденсатора (эта формула входит в программу школьного курса физики). При заряжении параллельных пластин, расположенных на небольшом расстоянии друг от друга, зарядами и , в пространстве между ними возникает однородное электрическое поле с напряженностью (см. гл. 18):

(20.2)

Разность потенциалов между пластинами равна

(20.3)

где — площадь пластин, — расстояние между ними. Отсюда, вычисляя отношение заряда к разности потенциалов (20.3), находим емкость плоского конденсатора

(20.4)

Если все пространство между обкладками заполнено диэлектриком с диэлектрической проницаемостью , то поле (20.2) и разность потенциалов (20.3) убывает в раз, а емкость конденсатора в раз взрастает

(20.5)

Для конденсаторов, соединенных в батареи, вводится понятие эквивалентной емкости, как емкости одного конденсатора, который при заряжении его тем же зарядом, что и батарея дает ту же разность потенциалов, что и батарея конденсаторов. Приведем формулы для эквивалентной емкости, а также для заряда и электрического напряжения на каждом конденсаторе при последовательном и параллельном их соединении.

Последовательное соединение (см. рисунок). При сообщении левой пластине левого конденсатора заряда , а правой пластине правого заряда , на внутренних пластинах благодаря поляризации будут индуцироваться заряды (см. рисунок; значения индуцированных зарядов приведены под пластинами). Можно доказать, что в результате поляризации каждый конденсатор будет заряжен такими же зарядами и , как и заряды крайних пластин, напряжение на всей батарее конденсаторов равно сумме напряжений на каждом, а обратная эквивалентная емкость батареи — сумме обратных емкостей всех конденсаторов

(20.6)

Параллельное соединение (см. рисунок). В этом случае если сообщить левому проводнику заряд , правому сообщить заряд , заряд распределится между конденсаторами, вообще говоря, не одинаково, но по закону сохранения заряда .

Поскольку правые пластины всех конденсаторов соединены между собой, левые — тоже, то они представляют собой единые проводники, и, следовательно, разность потенциалов между пластинами каждого конденсатора будет одинакова: . Можно доказать, что при таком соединении конденсаторов эквивалентная емкость батареи равна сумме емкостей отдельных конденсаторов

(20.7)

Заряженный конденсатор обладает определенной энергией. Если конденсатор емкости заряжен зарядом , то энергия этого конденсатора (можно говорить энергия электрического поля конденсатора) равна

(20.8)

С помощью определения электрической емкости (20.1) можно переписать формулу (20.8) еще в двух формах:

(20.9)

Рассмотрим в рамках этого минимума сведений о конденсаторах типичные задачи ЕГЭ по физике, которые были предложены в первой части книги.

Электроемкость конденсатора — его геометрическая характеристика, которая при неизменной геометрии не зависит от заряда конденсатора (задача 20.1.1 — ответ 3). Аналогично не меняется емкость конденсатора при увеличении напряжения на конденсаторе (задача 20.1.2 — ответ 3).

Связь между единицами измерений (задача 20.1.3) следует из определения емкости (20.1). Единица электрической емкости в международной системе единиц измерений СИ называется Фарада. 1 Фарада — это емкость такого конденсатора, между пластинами которого возникает напряжение 1 В при зарядах пластин 1 Кл и -1 Кл (ответ 4).

Поскольку электрическое поле в плоском конденсаторе однородно, то напряженность поля в конденсаторе и напряжение между пластинами связаны соотношением (см. формулу (18.9)) , где — расстояние между пластинами. Отсюда находим напряженность поля между обкладками плоского конденсатора в задаче 20.1.4

(ответ 4).

Согласно определению электрической емкости имеем в задаче 20.1.5

(ответ 2).

Из формулы (20.4) для емкости плоского конденсатора заключаем, что при увеличении площади его пластин в 3 раза (задача 20.1.6) его емкость увеличивается в 3 раза (ответ 1).

При уменьшении в раз расстояния между пластинами емкость плоского конденсатора возрастет в раз. Поэтому новое напряжение на конденсаторе (задача 20.1.7) можно найти из следующей цепочки формул

где и — новый заряд конденсатора (ответ 3).

Так как конденсатор в задаче 20.1.8 подключен к источнику, то между его пластинами поддерживается постоянное напряжение независимо от расстояния между ними. Поэтому заряд конденсатора изменяется при раздвигании пластин так же, как изменяется его емкость. А поскольку при увеличении расстояния между пластинами вдвое емкость конденсатора уменьшается вдвое (см. формулу (20.4)), то вдвое уменьшается и заряд конденсатора (ответ 2).

В задаче 20.1.9 конденсатор отключен от источника в процессе сближения пластин. Поэтому не меняется их заряд. А поскольку напряженность электрического поля между пластинами определяется соотношением (20.2)

то напряженность электрического поля между пластинами также не изменяется (ответ 3). Этот же результат можно получить и через определение емкости с учетом того, что

произведение от расстояния между пластинами не зависит (см. формулу (20.4)).

Из формул (20.8), (20.9) видим, что только одно из приведенных в качестве ответов к задаче 20.1.10 соотношений (а именно — 2) определяет энергию конденсатора.

При последовательном соединении конденсаторов (задача 20.2.1) одинаковыми будут их заряды независимо от значений их электрических емкостей (ответ 2). При параллельном соединении конденсаторов (задача 20.2.2) одинаковыми будут напряжения на каждом из них (ответ 3).

Поскольку конденсатор в задаче 20.2.3 отключен от источ-ника напряжения, его заряд не меняется в процессе раздвигания пластин. Поэтому для исследования изменения энергии конденсатора удобно воспользоваться формулой (20.8)

(1)

Так как при увеличении расстояния между пластинами в раз электрическая емкость конденсатора уменьшается в раз, то согласно формуле (1) энергия конденсатора увеличится в раз (ответ 1).

В задаче 20.2.4 не изменяется напряжение на конденсаторе. Поэтому воспользуемся первой из формул (20.9)

Из этой формулы заключаем, что при увеличении в раз расстояния между пластинами энергия конденсатора уменьшится в раз — ответ 2. (Разница с предыдущей задачей связана с тем, что здесь кроме внешних сил, совершающих работу при раздвигании пластин, совершает работу источник напряжения.)

В задаче 20.2.5 изменяют расстояние между пластинами (и, следовательно, емкость) и заряд конденсатора. Поэтому удобно воспользоваться формулой (20.8)

Из этой формулы заключаем, что при увеличении расстояния между пластинами в 2 раза и увеличении заряда конденсатора в 2 раза его энергия возрастет в 8 раз (ответ 4).

Поскольку в задаче 20.2.6 конденсаторы соединены последовательно, емкость батареи конденсаторов можно найти по формуле (20.6), откуда находим емкость батареи конденсаторов (ответ 2).

В задаче 20.2.7 конденсаторы соединены параллельно, поэтому емкость батареи конденсаторов можно найти по формуле (20.7): (ответ 2).

Основной вопрос, на который нужно ответить в задаче 20.2.8, это как соединены конденсаторы? Последовательно, параллельно, по-другому? Попробуем по-другому расположить в пространстве и изменить длину соединительных проводов, чтобы схема стала более понятной. Очевидно, что можно соединить вершину 1 и вершину 3 («уменьшив» длину провода 1-3), а также вершины 2 и 4. При этом средний конденсатор разворачивается в пространстве, и схема приобретает вид, показанный на рисунке, откуда видно, что конденсаторы соединены параллельно. Поэтому (ответ 1).

Когда в заряженный плоский конденсатор вставляют металлическую пластинку (задача 20.2.9), параллельную обкладкам конденсатора, напряженность электрического поля внутри пластинки становится равным нулю, вне пластинки между обкладками конденсатора остается таким же, каким оно было в отсутствие пластинки , где — заряд конденсатора, — площадь его пластин. Поэтому напряжение между обкладками конденсатора определяется соотношением:

где — расстояние между обкладками конденсатора, — толщина пластинки. Отсюда находим емкость рассматриваемого конденсатора

(ответ 4).

Чтобы найти емкость сферического конденсатора (задача 20.2.10) сообщим его обкладкам заряды и , найдем напряжение между обкладками, вычислим отношение заряда к напряжению. Разность потенциалов двух концентрических сфер, заряженных зарядами и (напряжение между обкладками сферического конденсатора), определена в задаче 19.2.5., откуда находим электрическую емкость сферического конденсатора (ответ 3):

Как изменяется электроемкость плоского конденсатора. Что такое электроемкость конденсатора? Электрическая емкость цилиндрического конденсатора

Одним их важнейших параметров, при помощи которого характеризуют конденсатор, является его электроёмкость (C). Физическая величина C, равная:

называется емкостью конденсатора. Где q — величина заряда одной из обкладок конденсатора, а — разность потенциалов между его обкладками. Электроемкость конденсатора — это величина, которая зависит то размеров и устройства конденсатора.

Для конденсаторов с одинаковым устройством и при равных зарядах на его обкладках разность потенциалов воздушного конденсатора будет в раз меньше, чем разность потенциалов между обкладками конденсатора, пространство которого между обкладками заполнено диэлектриком с диэлектрической проницаемостью . Значит емкость конденсатора с диэлектриком (C) в раз больше, чем электроемкость воздушного конденсатора ():

где — диэлектрическая проницаемость диэлектрика.

Единицей емкости конденсатора считают емкость такого конденсатора, который единичным зарядом (1 Кл) заряжается до разности потенциалов, равной одному вольту (в СИ). Единицей емкости конденсатора (как и любой эклектической емкости) в международной системе единиц (СИ) является фарад (Ф).

Электроемкость плоского конденсатора

Поле между обкладками плоского конденсатора в большинстве случаев считают однородным. Однородность нарушается только около краев. При расчете емкости плоского конденсатора данными краевыми эффектами обычно пренебрегают. Это возможно, если расстояние между пластинами мало в сравнении с их линейными размерами. В таком случае емкость плоского конденсатора вычисляют как:

где — электрическая постоянная; S — площадь каждой (или наименьшей) пластины; d — расстояние между пластинами.

Электрическая емкость плоского конденсатора, который содержит N слоев диэлектрика толщина каждого , соответствующая диэлектрическая проницаемость i-го слоя , равна:

Электрическая емкость цилиндрического конденсатора

Конструкция цилиндрического конденсатора включает две соосных (коаксиальных) цилиндрические проводящие поверхности, разного радиуса, пространство между которыми заполняет диэлектрик. Электрическая емкость такого конденсатора находят как:

где l — высота цилиндров; — радиус внешней обкладки; — радиус внутренней обкладки.

Емкости сферического конденсатора

Сферическим конденсатором называют конденсатор, обкладками которого являются две концентрические сферические проводящие поверхности, пространство между ними заполнено диэлектриком. Емкость такого конденсатора находят как:

где — радиусы обкладок конденсатора.

Примеры решения задач

ПРИМЕР 1

ЗаданиеПластины плоского воздушного конденсатора несут заряд, который равномерно распределен с поверхностной плотностью . При этом расстояние между его обкладками, равно . На какую величину изменится разность потенциалов на обкладках этого конденсатора, если его пластины раздвинуть до расстояния ?
РешениеСделаем рисунок.


В задаче при изменении расстояния между пластинами конденсатора заряд на его обкладках не изменяется, изменяются емкость и разность потенциалов на обкладках. Емкость плоского воздушного конденсатора равна:

где . Емкость этого же конденсатора можно определить как:

где U — разность потенциалов на обкладках конденсатора. Для конденсатора в первом случае имеем:

Для того же конденсатора, но после того как пластины раздвинули, имеем:

Используя формулу (1.3) и применяя соотношение:

выразим разность потенциалов

Следовательно, для конденсатора во втором состоянии получим:

Найдем изменение разности потенциалов:

Ответ

Рассмотрим два заряженных проводника. Предположим, что все силовые линии, начинающиеся на одном из них, заканчиваются на другом. Для этого, разумеется, они должны иметь равные и противоположные по знаку заряды. Такая система двух проводящих тел называется конденсатором.

Примеры конденсаторов. Примерами конденсаторов могут служить две концентрические проводящие сферы (сферический, или шаровой, конденсатор), две параллельные плоские проводящие пластины при условии, что расстояние между ними мало по сравнению с размерами пластин (плоский конденсатор), два коаксиальных проводящих цилиндра при условии, что их длина велика по сравнению с зазором между цилиндрами (цилиндрический конденсатор).

Два проводника, образующие конденсатор, называются его обкладками.

Рис. 41. Электрическое поле в сферическом, плоском и цилиндрическом конденсаторах

Во всех таких системах при сообщении обкладкам равных по модулю и противоположных по знаку зарядов электрическое поле практически целиком заключено в пространстве между обкладками (рис. 41). Внешний вид некоторых используемых в технике конденсаторов показан на рис. 42.

Основная характеристика конденсатора — электроемкость или просто емкость С, определяемая как отношение заряда одной из

обкладок к разности потенциалов т. е. к напряжению, между ними:

Распределение зарядов на обкладках будет одинаковым независимо от того, большой или малый заряд им сообщен. Это значит, что напряженность поля, а следовательно, и разность потенциалов между обкладками, пропорциональны сообщенному конденсатору заряду. Поэтому емкость конденсатора не зависит от его заряда.

Рис. 42. Устройство, внешний вид и условные обозначения на электрических схемах некоторых конденсаторов

В вакууме емкость определяется исключительно геометрическими характеристиками конденсатора, т. е. формой, размерами и взаимным расположением обкладок.

Единицы емкости. В СИ за единицу электроемкости принят фарад Емкостью 1 Ф обладает конденсатор, между обкладками которого устанавливается напряжение 1 В при сообщении заряда 1 Кл:

В абсолютной электростатической системе единиц СГСЭ электроемкость имеет размерность длины и измеряется в сантиметрах:

На практике обычно приходится иметь дело с конденсаторами, емкость которых значительно меньше 1 Ф. Поэтому используются доли этой единицы — микрофарад (мкФ) и пикофарад . Соотношение между фарадом и сантиметром легко установить, учитывая, что

Электроемкость и геометрия конденсатора. Зависимость емкости конденсатора от его геометрических характеристик легко проиллюстрировать простыми опытами. Воспользуемся для этого электрометром, подключенным к двум плоским пластинам, расстояние между которыми можно изменять (рис. 43). Чтобы заряды пластин были одинаковы и все поле было сосредоточено только между ними, следует заземлить вторую пластину и корпус электрометра. Отклонение стрелки электрометра пропорционально напряжению между обкладками. Если сдвигать или раздвигать пластины конденсатора, то при неизменном заряде напряжение будет соответственно уменьшаться или увеличиваться: емкость тем больше, чем меньше расстояние между пластинами. Аналогично можно убедиться в том, что емкость конденсатора тем больше, чем больше площадь его пластин. Для этого можно просто сдвигать пластины при неизменном зазоре между ними.

Рис. 43. Емкость конденсатора зависит от расстояния между пластинами

Емкость плоского конденсатора. Получим формулу для емкости плоского конденсатора. Поле между его обкладками однородно за исключением небольшой области вблизи краев пластин. Поэтому напряжение между обкладками равно произведению напряженности поля Е на расстоянии между ними: Для нахождения напряженности поля Е можно воспользоваться формулой (1) § 6, которая связывает Е вблизи поверхности проводника с поверхностной плотностью зарядов с: Выразим а через заряд конденсатора и площадь пластины, считая распределение заряда равномерным, что согласуется с используемым предположением об однородности поля: Подставляя приведенные соотношения в общее определение емкости (1), находим

В СИ, где емкость плоского конденсатора имеет вид

В системе единиц СГСЭ k = 1 и

Емкость сферического конденсатора. Совершенно аналогично можно вывести формулу для емкости сферического конденсатора, рассматривая электрическое поле в промежутке между двумя заряженными концентрическими сферами радиусов Напряженность поля там такая же, как в случае уединенного заряженного шара радиуса Поэтому для напряжения между обкладками радиусов справедливо

Выражение для емкости получаем, подставляя в формулу (1):

Емкость уединенного проводника. Иногда вводят понятие емкости уединенного проводника, рассматривая предельный случай конденсатора, одна из обкладок которого удалена на бесконечность. В частности, емкость уединенного проводящего шара получается из (5) в результате предельного перехода что соответствует неограниченному увеличению радиуса внешней обкладки при неизменном радиусе внутренней

В системе единиц СГСЭ, где емкость уединенного шара равна его радиусу. Если проводник имеет несферическую форму, его емкость по порядку величины равна характерному линейному размеру, хотя, конечно же, зависит и от его формы. В отличие от уединенного проводника, емкость конденсатора гораздо больше его линейных размеров. Например, у плоского конденсатора характерный линейный размер равен причем Как видно из формулы (4), при этом

Конденсатор с диэлектриком. В рассмотренных выше примерах конденсаторов пространство между обкладками считалось пустым. Тем не менее полученные выражения для емкости справедливы и тогда, когда это пространство заполнено воздухом, как это было в описанных простых опытах. Если пространство между обкладками заполнить каким-либо диэлектриком, емкость конденсатора увеличивается. В этом легко убедиться на опыте, вдвигая диэлектрическую пластину в промежуток между обкладками заряженного конденсатора, подключенного к электрометру (рис. 43). При неизменном заряде конденсатора напряжение между обкладками уменьшается, что свидетельствует о возрастании емкости.

Уменьшение разности потенциалов между обкладками при внесении туда диэлектрической пластины свидетельствует о том, что напряженность электрического поля в зазоре становится меньше. Это уменьшение зависит от того, какой именно диэлектрик используется в опыте.

Диэлектрическая проницаемость. Для характеристики электрических свойств диэлектрика вводят физическую величину, называемую диэлектрической проницаемостью. Диэлектрическая проницаемость — это безразмерная величина, показывающая, во сколько раз напряженность электрического поля в заполненном диэлектриком конденсаторе (или напряжение между его обкладками) меньше, чем в отсутствие диэлектрика при том же заряде конденсатора. Другими словами, диэлектрическая проницаемость показывает, во сколько раз увеличивается емкость конденсатора при заполнении его диэлектриком. Например, емкость плоского конденсатора, заполненного диэлектриком с проницаемостью равна

Приведенное здесь определение диэлектрической проницаемости соответствует феноменологическому подходу, при котором рассматриваются только макроскопические свойства вещества в электрическом поле. Микроскопический подход, основанный на рассмотрении поляризации атомов или молекул, из которых состоит вещество, предполагает исследование какой-либо конкретной модели и позволяет не только подробно описывать электрические и магнитные поля внутри вещества, но и понять, как протекают макроскопические электрические и магнитные явления в веществе. На этом этапе мы ограничиваемся только феноменологическим подходом.

Рис. 44. Параллельное соединение конденсаторов

У твердых диэлектриков значение лежит в пределах от 4 до 7, а у жидких — от 2 до 81. Такой аномально большой диэлектрической проницаемостью обладает обыкновенная чистая вода. Кроме воздушного конденсатора переменной емкости (см. рис. 42), используемого для настройки радиоприемников, все другие применяемые в технике конденсаторы заполнены диэлектриком.

Батареи конденсаторов. При использовании конденсаторов их иногда соединяют в батареи. При параллельном соединении (рис. 44) напряжения на конденсаторах одинаковы, а полный заряд батареи равен сумме зарядов конденсаторов для каждого из которых, очевидно, справедливо Рассматривая батарею как один

конденсатор, имеем

С другой стороны,

Сравнивая (8) и (9), получаем, что емкость батареи параллельно соединенных конденсаторов равна сумме их емкостей:

Рис. 45. Последовательное соединение конденсаторов

При последовательном соединении предварительно незаряженных конденсаторов (рис. 45) заряды на всех конденсаторах одинаковы, а полное напряжение равно сумме напряжений на отдельных конденсаторах:

С другой стороны, рассматривая батарею как один конденсатор, имеем

Сравнивая (11) и (12), видим, что при последовательном соединении конденсаторов складываются обратные емкостям величины:

При последовательном соединении емкость батареи меньше самой малой из емкостей соединенных конденсаторов.

В каком случае два проводящих тела образуют конденсатор?

Что называется зарядом конденсатора?

Как установить связь между единицами емкости СИ и СГСЭ?

Объясните качественно, почему емкость конденсатора увеличивается при уменьшении зазора между обкладками.

Получите формулу для емкости плоского конденсатора, рассматривая электрическое поле в нем как суперпозицию полей, создаваемых двумя плоскостями, заряженными разноименно.

Получите формулу для емкости плоского конденсатора, рассматривая его как предельный случай сферического конденсатора, у которого стремятся к бесконечности так, что разность остается постоянной.

Почему нельзя говорить о емкости уединенной бесконечной плоской пластины или отдельного бесконечно длинного цилиндра?

Охарактеризуйте кратко различие между феноменологическим и микроскопическим подходами при исследовании свойств вещества в электрическом поле.

Каков смысл диэлектрической проницаемости вещества?

Почему при расчете емкости батареи последовательно соединенных конденсаторов оговаривалось условие, чтобы они предварительно не были заряжены?

В чем смысл последовательного соединения конденсаторов, если оно приводит лишь к уменьшению емкости?

Поле внутри и вне конденсатора. Чтобы подчеркнуть различие между тем, что называют зарядом конденсатора, и полным зарядом обкладок, рассмотрим следующий пример. Пусть наружная обкладка сферического конденсатора заземлена, а внутренней сообщен заряд д. Весь этот заряд равномерно распределится по внешней поверхности внутренней обкладки. Тогда на внутренней поверхности наружной сферы индуцируется заряд , следовательно, заряд конденсатора равен . А что будет на внешней поверхности наружной сферы? Это зависит от того, что окружает конденсатор. Пусть, например, на расстоянии от поверхности внешней сферы находится точечный заряд (рис. 46). Этот заряд никак не повлияет на электрическое состояние внутреннего пространства конденсатора, т. е. на поле между его обкладками. В самом деле, внутреннее и внешнее пространства разделены толщей металла наружной обкладки, в которой электрическое поле равно нулю.

Рис. 46. Сферический конденсатор во внешнем электрическом поле

Заряд на внешней поверхности обкладки. Но характер поля во внешнем пространстве и заряд, индуцированный на наружной поверхности внешней сферы, зависят от величины и положения заряда Это поле будет точно таким же, как и в случае, коща заряд находится на расстоянии от поверхности сплошного заземленного металлического шара, радиус которого равен радиусу внешней сферы конденсатора (рис. 47). Таким же будет и индуцированный заряд.

Для нахождения величины индуцированного заряда будем рассуждать следующим образом. Электрическое поле в любой точке пространства создается зарядом и зарядом, индуцированным

на поверхности шара, который распределен там, разумеется, неравномерно — как раз так, чтобы обратилась в нуль результирующая напряженность поля внутри шара. Согласно принципу суперпозиции потенциал в любой точке можно искать в виде суммы потенциалов полей, создаваемых точечным зарядом и точечными зарядами, на которые можно разбить распределенный по поверхности шара индуцированный заряд. Поскольку все элементарные заряды на которые разбит индуцированный на поверхности шара заряд находятся на одинаковом расстоянии от центра шара, то потенциал создаваемого им поля в центре шара будет равен

Рис. 47. Поле точечного заряда вблизи заземленного проводящего шара

Тогда полный потенциал в центре заземленного шара равен

Знак минус отражает тот факт, что индуцированный заряд всегда противоположного знака.

Итак, мы видим, что заряд на наружной поверхности внешней сферы конденсатора определяется тем окружением, в котором находится конденсатор, и не имеет никакого отношения к заряду конденсатора д. Полный заряд внешней обкладки конденсатора, разумеется, равен сумме зарядов ее внешней и внутренней поверхностей, однако заряд конденсатора определяется только зарядом внутренней поверхности этой обкладки, который связан силовыми линиями поля с зарядом внутренней обкладки.

В разобранном примере независимость электрического поля в пространстве между обкладками конденсатора и, следовательно, его емкости от внешних тел (как заряженных, так и незаряженных) обусловлена электростатической защитой, т. е. толщей металла внешней обкладки. К чему может привести отсутствие такой защиты, можно увидеть на следующем примере.

Плоский конденсатор с экраном. Рассмотрим плоский конденсатор в виде двух параллельных металлических пластин, электрическое поле которого практически целиком сосредоточено в пространстве между пластинами. Заключим конденсатор в незаряженную плоскую металлическую коробку, как показано на рис. 48. На первый взгляд может показаться, что картина поля между обкладками конденсатора не изменится, так как все поле сосредоточено между пластинами, а краевым эффектом мы пренебрегаем. Однако легко видеть, что это не так. Снаружи конденсатора напряженность поля равна нулю, поэтому во всех точках слева от конденсатора потенциал одинаков и совпадает с потенциалом левой пластины. Точно так же потенциал любой точки справа от конденсатора совпадает с потенциалом правой пластины (рис. 49). Поэтому, заключая конденсатор в металлическую коробку, мы соединяем проводником точки, имеющие разный потенциал.

В результате в металлической коробке будет происходить перераспределение зарядов до тех пор, пока не выравняются потенциалы всех ее точек. На внутренней поверхности коробки индуцируются заряды, и появится электрическое поле внутри коробки, т. е. снаружи конденсатора (рис. 50).

Рис. 48. Конденсатор в металлической коробке

Рис. 49. Электрическое поле заряженного плоского конденсатора

Рис. 50. Электрическое поле заряженного конденсатора, помещенного в металлическую коробку

Но это означает, что на внешних поверхностях пластин конденсатора тоже появятся заряды. Так как при этом полный заряд изолированной пластины не меняется, то заряд на ее внешней поверхности может возникнуть только за счет перетекания заряда с внутренней поверхности. Но при изменении заряда на внутренних поверхностях обкладок изменится напряженность поля между пластинами конденсатора.

Таким образом, заключение рассмотренного конденсатора в металлическую коробку приводит к изменению электрического состояния внутреннего пространства.

Изменение зарядов пластин и электрического поля в этом примере может быть легко рассчитано. Обозначим заряд изолированного конденсатора через Заряд, перетекающий на наружные поверхности пластин при надевании коробки, обозначим через Такой же заряд противоположного знака будет индуцирован на внутренних поверхностях коробки. На внутренних поверхностях пластин конденсатора останется заряд Тогда в пространстве между пластинами напряженность однородного поля будет равна в единицах СИ, а вне конденсатора поле направлено в противоположную сторону и его напряженность равна где — площадь пластины. Требуя, чтобы разность потенциалов между противоположными стенками металлической коробки была равна нулю, и считая для простоты расстояния между всеми пластинами одинаковыми и равными то

Этот результат легко понять, если учесть, что после надевания коробки поле существует во всех трех промежутках между пластинами, т. е. фактически имеются три одинаковых конденсатора, эквивалентная схема включения которых показана на рис. 51. Вычисляя емкость получившейся системы конденсаторов, получаем .

Надетая на конденсатор металлическая коробка осуществляет электростатическую защиту системы. Теперь мы можем подносить снаружи к коробке любые заряженные или незаряженные тела и при этом электрическое поле внутри коробки не изменится. Значит, не изменится и емкость системы.

Обратим внимание на то, что в разобранном примере, выяснив все, что нас интересовало, мы тем не менее обошли стороной вопрос о том, какие же силы осуществили перераспределение зарядов. Какое электрическое поле вызвало движение электронов в материале проводящей коробки?

Очевидно, что это может быть только то неоднородное поле, которое выходит за пределы конденсатора вблизи краев пластины (см. рис. 39). Хотя напряженность этого поля мала и не принимается во внимание при расчете изменения емкости, именно она определяет суть рассматриваемого явления — перемещает заряды и этим вызывает изменение напряженности электрического поля внутри коробки.

Почему под зарядом конденсатора следует понимать не полный заряд обкладки, а только ту его часть, что находится на ее внутренней стороне. обращенной к другой обкладке?

В чем проявляется роль краевых эффектов при рассмотрении электростатических явлений в конденсаторе?

Как изменится емкость батареи конденсаторов, если замкнуть между собой обкладки одного из них?

Формула электроемкости следующая.

Измеряется эта величина в фарадах. Как правило, емкость элемента очень мала и измеряется в пикофарадах.

В задачах часто спрашивается, как изменится электроемкость конденсатора, если увеличить заряд или напряжение. Это вопрос с подвохом. Проведем другую аналогию.

Представьте, что речь идет про обычную банку, а не конденсатор. Например, у вас она трехлитровая. Аналогичный вопрос: что произойдет со вместимостью банки, если туда налить 4 литра воды? Разумеется, вода просто выльется, но при этом размеры банки никак не изменятся.

То же самое с конденсаторами. Заряд и напряжение никак не влияют на емкость. Этот параметр зависит только от реальных физических размеров.

Формула будет следующей

Только эти параметры влияют на реальную электроемкость конденсатора.

На любом конденсаторе есть маркировка с техническими параметрами.

Разобраться несложно. Достаточно минимальных знаний по электричеству.

Соединение конденсаторов

Конденсаторы, так же как и сопротивления, можно подключать последовательно и параллельно. Кроме этого, в схемах бывают и смешанные соединения.

Как видите, электроемкость конденсатора в обоих случаях считается по-разному. Это также относится к напряжению и заряду. По формулам видно, что электроемкость конденсатора, вернее, их совокупности в схеме, будет наибольшей при параллельном соединении. При последовательном общая емкость значительно уменьшается.

При подключении последовательно заряд размещается равномерно. Он будет везде одинаков — как суммарный, так и на каждом конденсаторе. А когда соединение параллельное, суммарный заряд складывается. Это важно помнить при решении задач.

Напряжение считается наоборот. При последовательном соединении складываем, а при параллельном оно равно везде.

Здесь приходится выбирать: если вам нужно больше напряжения, тогда жертвуем емкостью. Если емкость, то огромного напряжения не будет.

Виды конденсаторов

Существует огромное количество конденсаторов. Они отличаются как по размеру, так и по форме.

Разумеется, емкость вычисляется у всех по-разному.

Электроемкость плоского конденсатора

Электроемкость плоского конденсатора определяется проще всего. Эту формулу в основном все и помнят, в отличии от других.

Здесь всё зависит от физических параметров и среды между пластинами.

Здесь также большое значение имеет, какой диэлектрик или материал помещен внутрь. Так как деталь имеет размер сферы, ее емкость зависит от радиуса.

В случае с цилиндрической формой, кроме среды внутри, значение имеют радиусы и длина цилиндра.

Подумайте, как изменится электроемкость плоского конденсатора, если на нем будут повреждения? Существуют различные сбои, которые могут повлиять на работоспособность конденсаторов.

Например, они рассыхаются или вздуваются. После этого они становятся непригодными для нормальной работы устройства, куда установлены.

Рассмотрим примеры повреждений и выхода из строя конденсаторов. Вздуться могут все сразу.

Иногда из строя выходят только несколько. Такое бывает, когда конденсаторы разных параметров или качества.

Наглядный пример порчи (вздутие, разрыв и выход наружу содержимого).

Если вы увидите вот такие ленты, это крайняя степень повреждения. Хуже и быть не может.

Если вы заметите на устройстве (например на видеокарте в компьютере) такие вздутые конденсаторы, это повод задуматься о замене детали.

Подобные проблемы можно устранить только заменой на аналогичную деталь. У вас должны совпадать все параметры один в один. Иначе работа может быть некорректной или очень кратковременной.

Менять конденсаторы нужно аккуратно, не повредив платы. Выпаивать нужно быстро, не допуская перегрева. Если вы не умеете этого делать, лучше отнесите деталь в ремонт.

Основной причиной разрушения является перегрев, который возникает в случае старения или большого сопротивления в цепи.

Рекомендуется не затягивать с ремонтом. Поскольку у поврежденных конденсаторов изменяется емкость, устройство, где они расположены, будет работать с отклонением от нормы. И со временем это может стать причиной выхода из строя.

Если у вас на видеокарте вздулись конденсаторы, то их своевременная замена может исправить ситуацию. В противном случае может сгореть микросхема или что-то еще. В таком случае ремонт будет стоить очень дорого или вовсе окажется невозможным.

Меры предосторожности

Выше был приведен пример с банкой воды. Там говорилось, что если воды налить больше, то воды выльется. А теперь подумайте, куда могут «вылиться» электроны в конденсаторе? Ведь он запечатан полностью!

Если вы подадите в цепи больше тока, чем тот, на который рассчитан конденсатор, то как только он зарядится, его излишек попытается выйти куда-то. А пространства свободного нет. Результатом будет взрыв. В случае незначительного превышения заряда хлопок будет небольшой. Но если подать колоссальное количество электронов на конденсатор, его просто разорвет, и диэлектрик вытечет.

Будьте аккуратны!

Плоским конденсатором обычно называ-ют систему плоских проводящих пластин — обкладок, разделенных диэлектриком. Про-стота конструкции такого конденсатора по-зволяет сравнительно просто рассчитывать его электроемкость и получать значения, совпадающие с результатами эксперимента.

Укрепим две металлические пластины на изоляционных подставках и соединим с электрометром так, что одна из пластин будет присоединена к стержню электромет-ра, а вторая — к его металлическому кор-пусу (рис. 4.71). При таком соединении электрометр будет измерять разность по-тенциалов между пластинами, которые об-разуют плоский конденсатор из двух пла-стин. Проводя исследования, необходимо пом-нить, что

при постоянном значении заряда пластин уменьшение разности потенциалов свидетельствует об увеличении электроем-кости конденсатора, и наоборот.

Сообщим пластинам разноименные заря-ды и отметим отклонение стрелки электро-метра. Приближая пластины друг к другу (уменьшая расстояние между ними), заме-тим уменьшение разности потенциалов. Та-ким образом, при уменьшении расстояния между пластинами конденсатора его элект-роемкость увеличивается. При увеличении расстояния показания стрелки электрометра увеличиваются, что является свидетельст-вом уменьшения электроемкости.

об-ратно пропорциональна расстоянию между его обкладками.

C ~ 1 / d ,

где d — расстояние между обкладками.

Эту зависимость можно изобразить гра-фиком обратной пропорциональной зависи-мости (рис. 4.72).

Будем смещать пластины одну относи-тельно другой в параллельных плоскостях, не изменяя расстояния между ними.

При этом площадь перекрытия пластин будет уменьшаться (рис. 4.73). Увеличение разности потенциалов, отмеченное электрометром, будет свидетельствовать об умень-шении электроемкости.

Увеличение площади перекрытия пластан приведет к увеличению емкости.

Электроемкость плоского конденсатора про-порциональна площади пластин, которые пере-крываются.

C ~ S,

где S — площадь пластин.

Эту зависимость можно представить гра-фиком прямой пропорциональной зависи-мости (рис. 4.74).

Возвратив пластины в начальное поло-жение, внесем в пространство между ними плоский диэлектрик. Электрометр отметит уменьшение разности потенциалов между пластинами, что свидетельствует об увели-чении электроемкости конденсатора. Если между пластинами поместить другой диэлек-трик, то изменение электроемкости будет иным.

Электроемкость плоского конденсатора за-висит от диэлектрической проницаемости ди-электрика.

C ~ ε ,

где ε — диэлектрическая проницаемость ди-электрика. Материал с сайта

Такая зависимость показана на графике рис. 4.75.

Результаты опытов можно обобщить в ви-де формулы ёмкости плоского конденсатора :

C = εε 0 S / d,

где S — площадь пластины; d — расстояние между ними; ε — диэлектрическая прони-цаемость диэлектрика; ε 0 — электрическая постоянная.

Конденсаторы, которые состоят из двух пластин, в практике применяются очень редко. Как правило, конденсаторы имеют много пластин, соединенных между собой по определенной схеме.

На этой странице материал по темам:

  • Решение задач по теме электроемкость плоского конденсатора

  • Как влияет диэлектрик на электроёмкость?

  • Теория плоских конденсаторов

  • График электроемкости плоского конденсатора от площади его пластин

  • Заключение по электроемкости

Вопросы по этому материалу:

  • Какое строение плоского конденсатора?

  • По изменению какой величины в опыте можно делать заключение об изменении электроемкости?

ESR конденсатора — что это?

ESR — Equivalent Series Resistance — один из параметров конденсатора, характеризующий его активные потери в цепи переменного тока. В эквиваленте его можно представить, как включенный последовательно с конденсатором резистор, сопротивление которого определяется, главным образом, диэлектрическими потерями, а так же сопротивлением обкладок, внутренних контактных соединений и выводов. В русскоязычной аббревиатуре — Эквивалентное Последовательное Сопротивление — ЭПС.

Потери в диэлектрике, обусловленные особенностями его поляризации, составляют основную часть потерь в конденсаторе и определяются материалом, а так же толщиной слоя диэлектрика.

Поляризация — ограниченное смещение связанных зарядов диэлектрика в электрическом поле.

Рассматривать детально процессы всех видов поляризации здесь нет необходимости, но вкратце это можно пояснить следующим образом:
Частицы диэлектрика, обладающие зарядом, под воздействием переменного электрического поля вынуждены совершать непроизвольные механические колебания, обусловленные их переориентацией и смещением (поляризацией).
В слоях диэлектрика, близких к обкладкам, заряды, не покидая своих связей, активно участвуют во всех процессах формирования напряжения и тока в конденсаторе, как и проводники. По сути, уменьшается толщина слоя реального диэлектрика.
В результате существенно повышается ёмкость конденсатора но, по причине инертности и внутреннего трения связанных частиц, процессы сопровождаются выделением тепла и потерями энергии в токопроводящих слоях диэлектрика. То есть, эти поляризованные слои обладают активным сопротивлением электрическому току.
С увеличением частоты, диэлектрические потери пропорционально возрастают по той же причине — механической инертности поляризованных зарядов.

Сопротивление токопроводящих слоёв диэлектрика последовательно складывается с сопротивлением обкладок, выводов и контактных соединений. В итоге образуется общее активное сопротивление R — Equivalent Series Resistance (ESR). По сути оно представляет собой резистор, включенный последовательно с конденсатором.

В этом случае угол сдвига фаз между током и напряжением будет не 90°, как в идеальном конденсаторе, а несколько меньше.
Тангенс угла δ, составляющего эту разницу с 90°, называют тангенсом угла потерь.

Тангенс угла определится отношением активного сопротивления к реактивному R/Xc, как тригонометрическая функция отношения двух катетов треугольника сопротивлений, показанного на рисунке выше.

В электролитических конденсаторах значимой частью ESR является сопротивление жидкого электролита, который используется в качестве одной из обкладок для обеспечения максимальной площади соприкосновения с диэлектриком.
Активное сопротивление электролита в реальных конденсаторах обычно соизмеримо с десятыми или даже с сотыми долями Ома при 20°C, но для конденсаторов большой ёмкости, используемых в фильтрах выпрямителей ИИП на рабочей частоте порядка 100 кГц, когда его реактивное сопротивление измеряется тысячными долями Ома, эта величина может составлять основные потери, и будет значительно уменьшаться по мере прогрева.
При рабочей температуре величина диэлектрических потерь на таких частотах обычно оказывается в несколько раз больше.

Сопротивление электролита зависит от температуры по причине изменения степени его вязкости и подвижности ионов.

В процессе работы происходит нагрев диэлектрика и электролита переменным током, в связи с чем существенно уменьшается сопротивление электролита, тогда ESR конденсатора будет определяться преимущественно его диэлектрическими потерями, которые продолжат греть конденсатор в допустимых расчётами пределах.
Но, в случаях разогрева до температуры кипения, электролит утрачивает свои первоначальные свойства и при последующем охлаждении становится более вязким, что ухудшает подвижность ионов и повышает активное сопротивление. Дальнейшая эксплуатация будет вызывать ещё больший разогрев и ухудшение качества электролита, что в последствии приведёт к непригодности конденсатора для дальнейшей работы.
Неисправные конденсаторы, в которых кипел электролит, обычно определяются визуально по вздувшемуся и разгерметизированному корпусу.

Для надёжности работы электролитических конденсаторов очень важен правильный выбор его типа, номинала и максимального напряжения в зависимости от режимов и условий эксплуатации.
Для фильтров выпрямителей в преобразователях, работающих на частотах десятков или сотен килогерц, производители выпускают специальные конденсаторы с малым ESR и указывают полное сопротивление переменному току (импеданс Z) для всех номиналов в таблицах.
Тип таких конденсаторов сопровождается пометкой в технической документации — Low impedance или Low ESR.

Для анализа состояния электролита и внутренних соединений электролитических конденсаторов применяются измерители или пробники ESR, которые могут быть выполнены исходя из разных принципов измерений и требований к погрешностям.
Большая часть простых ESR-пробников и тестеров основана на принципе измерения импеданса. У них есть свой существенный плюс — низкоомный вход, что позволяет проверять конденсаторы, не выпаивая их из платы.
Подробнее о способах измерения можно ознакомиться на страничке — измерение ESR.

Наряду с ухудшением качества электролита, часто активное сопротивление в конденсаторах возрастает по причине ухудшения контактов обкладок с выводами, вплоть до полного обрыва. В электролитических это происходит чаще, в металлокерамических реже, телевизионным мастерам все эти случаи хорошо знакомы. А ремонтники старшего поколения, кто застал советские ламповые телевизоры, хорошо помнят бумажные конденсаторы, которые иногда поджимали пассатижами для уплотнения контактных соединений внутри, и они какое-то время ещё работали.

Для чего нужна таблица?
Большинство пробников и тестеров, обычно светодиодные или стрелочные, измеряют импеданс — общее сопротивление конденсатора (активное и реактивное). Активное отдельно замерить сложнее, но оно и есть потери — значение ESR.
При измерении ёмкостей менее 100 микрофарад, реактивная составляющая уже оказывается соизмеримой, а иногда больше значения ESR, и существенно влияет на результат. А в конденсаторах менее 10 мкф и вовсе значение ESR во много раз меньше и его доля незначительна в общем показании. Точно замерить ESR у них невозможно такими пробниками, но выявить неисправные конденсаторы можно.
Другими словами, реактивное сопротивление в показаниях таких приборов — неудобная погрешность, зависимая от ёмкости конденсатора. Её надо учитывать при оценке качества конденсатора для разных ёмкостей.
К тому же ESR зависит от толщины слоя электролита и диэлектрика. Для высоковольтных и крупногабаритных конденсаторов эти значения учитываются производителями в зависимости от области применения.
Никакой пропорциональной зависимости ESR от других параметров конденсатора не существует, поэтому для оценки его качества в практике используются таблицы.

Все существующие таблицы — условны и не всегда объективно определяют допустимые значения для всех измерителей. Публикуют их часто для популяризации сайтов, поэтому важно понимать суть значений в таблицах.
Тем более, разные пробники работают на разных принципах или частотах (от 10 до 100 кГц), разница показаний в 5 или 10 раз может отличаться от табличных лишь по этой причине.
Очень полезно самому замерить значения ESR у новых конденсаторов разных производителей и составить свою таблицу для своего пробника. Это уже будут реальные показатели. Тогда их можно сравнить с неисправными конденсаторами и со значениями их реактивных сопротивлений, чтоб сделать какие-то выводы о критичности.
В преобразователях блоков питания греют конденсатор паразитные десятые, иногда сотые доли Ома и, если их сможет показать Ваш измеритель, уже неплохо. Импульсный ток в конденсаторах достигает десятков Ампер и активные десятые доли Ома для 10 Ампер — это уже реальные Ватты — нагрев.
Габариты конденсатора тоже имеют существенное значение, они будут охлаждать электролит, это надо учитывать при выборе типа конденсатора в мощных преобразователях.
Практика показала, тонкие конденсаторы Low ESR, установленные при замене в блоках питания вместо крупногабаритных обычных, частенько долго там не живут, перегреваются, закипают и вздуваются иногда уже через несколько месяцев работы.

Для самого популярного в ИИП конденсатора 1000мкф x 25в часто в таблицах указывают 0.08 Ом, как норму. А в других таблицах 0.8 Ом. Какой прибор что мерит, кто и для каких цепей определил ему норму — загадки.
Проверьте для сравнения своим прибором этот конденсатор новый от разных производителей, в том числе с пометкой Low ESR, тогда оценка будет объективнее.

Таблица Боба Паркера для ESR-метра K7214

uF\V10V16V25V35V50V160V250V
1 uF14161820
2.2 uF68101010
4.7 uF157.54.22.35
10 uF643.52.435
22uF5.43.62.11.51.51.53
47 uF2.21.61.20.50.50.70.8
100 uF1.20.70.320.320.30.150.8
220 uF0.60.330.230.170.160.090.5
470 uF0.240.20.150.10.10.10.3
1000 uF0.120.10.080.070.050.06
4700 uF0.230.20.120.060.06

Рассчитаем округлённо реактивное сопротивление для популярных номиналов при усреднённой частоте пробников 20 кГц, чтобы иметь представление хотя бы о порядке их идеальных значений.

Ещё раз напомню, никакой пропорции между ESR и этими значениями быть не может. Тем более, с учётом конструктивных особенностей электролитических конденсаторов для разных габаритов и вольтажа.
Повторюсь. Это лишь реактивное сопротивление, которое имеет большее значение при измерении конденсаторов меньшей ёмкости, как реальная погрешность для пробников, основанных на измерении импеданса.
То есть, чистое значение ESR у конденсатора 100 мкф и 1 мкф может быть одинаковым, а прибор покажет разницу в десятки раз, ибо добавит ёмкостное значение, которое будет решающим для показаний прибора на измеряемой частоте у малых ёмкостей.

Реактивное сопротивление конденсаторов, частота 20кГц:
1000 мкф — 0.008 Ом.
470 мкф — 0.017 Ом.
220 мкф — 0.036 Ом.
100 мкф — 0.08 Ом.
47 мкф — 0.17 Ом.
22 мкф — 0.36 Ом.
10 мкф — 0.8 Ом.
4.7 мкф — 1.7 Ом.
2.2 мкф — 3.6 Ом.
1 мкф — 8 Ом.
0.47 мкф — 17 Ом.
Поможет калькулятор расчёта реактивного сопротивления конденсаторов.

Более сложные цифровые приборы способны замерить точные значения во время заряда конденсатора постоянным током, рассчитать его ёмкость и ESR без реактивной составляющей.
Но измерение постоянным током не учитывает диэлектрические потери, которые напрямую зависят от частоты. Кроме того, конденсаторы нужно выпаивать из платы для таких замеров.

Пробниками обычно быстро проверяют конденсаторы на неисправность, не выпаивая их, а это существенный выигрыш в оперативности для мастера — ремонтника. Ему не всегда нужны точные показания сложных приборов, чаще бывает важно своевременно и правильно выявить неисправную деталь в устройстве. К погрешностям на реактивность в практике мастера просто привыкают, когда годами пользуются одним и тем же пробником.

Спасибо за внимание!


Замечания и предложения принимаются и приветствуются!

Конденсаторы для асинхронных двигателей | Насосы и принадлежности

Добрый день, уважаемые читатели блога nasos-pump.ru

Конденсаторы

В рубрике «Принадлежности» рассмотрим конденсаторы для однофазных асинхронных двигателей переменного тока. У трехфазных двигателей при подключении к сети питания возникает вращающееся магнитное поле, за счет которого и происходит запуск двигателя. В отличие от трехфазных двигателей, у однофазных в статоре имеется две обмотки рабочая и пусковая. Рабочая обмотка подключена к однофазной сети питания напрямую, а пусковая последовательно с конденсатором. Конденсатор необходим для создания сдвига фаз между токами рабочей и пусковой обмоток. Самый большой вращающий момент в двигателе возникает тогда, когда сдвиг фаз токов обмоток достигает 90°, а их амплитуды создают круговое вращающееся поле. Конденсатор является элементом электрической цепи и предназначен для использования его ёмкости. Он состоит из двух электродов или правильней обкладок, которые разделёны диэлектриком. Конденсаторы имеют возможность накапливать электрическую энергию. В Международной системе единиц СИ за единицу ёмкости принимается ёмкость конденсатора, у которого на один вольт возрастает разность потенциалов при сообщении ему заряда в один кулон (Кл). Емкость конденсаторов измеряется в фарадах (Ф). Емкость в одну фараду очень большая. На практике используются более мелкие единицы измерения микрофарады (мкФ) одна мкФ равняется 10-6 Ф, пикофарады (пФ) одна пФ равняется 10-12 мкФ. В однофазных асинхронных двигателях в зависимости от мощности используются конденсаторы емкостью от нескольких до сотен мкФ.

Основные электрические параметры и характеристики

К основным электрическим параметрам конденсаторов для асинхронных двигателей относятся: номинальная емкость конденсатора и номинальное рабочее напряжение. Кроме этих параметров существует еще температурный коэффициент емкости (ТКЕ), тангенс угла потерь (tgd), электрическое сопротивление изоляции.

Емкость конденсатора. Свойство конденсатора накапливать и удерживать электрический заряд характеризуется его емкостью. Емкость (С) определяется как отношение накопленного в конденсаторе заряда (q), к разности потенциалов на его электродах или приложенному напряжению (U). Емкость конденсаторов зависит от размеров и формы электродов, их расположения друг относительно друга, а также материала диэлектрика который разделяет электроды. Чем емкость конденсатора больше, тем и накопленный им заряд больше Удельная ёмкость конденсатора – выражает отношение его ёмкости к объёму. Номинальная ёмкость конденсатора – это ёмкость, которую имеет конденсатор согласно нормативной документации. Фактическая же ёмкость каждого отдельного конденсатора отличается от номинальной, но она должна быть в пределах допускаемых отклонений. Значения номинальной ёмкости и ее допустимое отклонение в различных типах конденсаторов постоянной ёмкости установлена стандартом.

Номинальное напряжение – это то значение напряжения обозначенное на конденсаторе, при котором он работает в заданных условиях длительное время и при этом сохраняет свои параметры в допустимых пределах. Значение номинального напряжения зависит от свойств используемых материалов и конструкции конденсаторов. В процессе эксплуатации рабочее напряжение на конденсаторе не должно превышать номинальное. У многих типов конденсаторов при увеличении температуры допустимое номинальное напряжение снижается.

Температурный коэффициент емкости (ТКЕ) – это параметр выражающий линейную зависимостью емкости конденсатора от температуры внешней среды. На практике ТКЕ определятся как относительное изменение емкости при изменении температуры на 1°С. Если эта зависимость нелинейная, то ТКЕ конденсатора характеризуется относительным изменением емкости при переходе от нормальной температуры (20±5°С) к допустимому значению рабочей температуры. Для конденсаторов используемых в однофазных двигателях этот параметр важный и должен быть как можно меньше. Ведь в процессе эксплуатации двигателя его температура повышается, а конденсатор находится непосредственно на двигателе в конденсаторной коробке.

Тангенс угла потерь (tgd). Потеря накопленной энергии в конденсаторе обусловлена потерями в диэлектрике и его обкладках. Когда через конденсатор протекает переменный ток, то векторы тока и напряжения сдвинуты относительно друг друга на угол (d). Этот угол (d) и называют углом диэлектрических потерь. Если потери отсутствуют, то d=0. Тангенс угла потерь это отношение активной мощности (Pа) к реактивной (Pр) при напряжении синусоидальной формы определённой частоты.

Электрическое сопротивление изоляции – электрическое сопротивление постоянному току, определяется как отношение приложенного к конденсатору напряжения (U) , к току утечки (Iут), или проводимости. Качество применяемого диэлектрика и характеризует сопротивление изоляции. Для конденсатора с большой емкостью сопротивление изоляции обратно пропорционально его площади обкладок, или его ёмкости.

На конденсаторы оказывает очень сильное воздействие влага. Асинхронные электродвигатели используемые в насосном оборудовании перекачивают воду, и высока вероятность попадания влаги на двигатель и в конденсаторную коробку. Воздействие влаги приводит к снижению сопротивления изоляции (возрастает вероятность пробоя), увеличению тангенса угла потерь, коррозии металлических элементов конденсатора.

Кроме всего при эксплуатации двигателя на конденсаторы воздействует различного вида механические нагрузки: вибрация, удары, ускорение и т.д. Как следствие могут появится обрыв выводов, трещины и уменьшение электрической прочности.

Рабочий и пусковой конденсаторы

В качестве рабочих и пусковых используются конденсаторы с оксидным диэлектриком (ранее они назвались электролитическими) Рабочие и пусковые конденсаторы для асинхронных двигателей включаются в сеть переменного тока, и они должны быть неполярными. Они имеют сравнительно большое 450 вольт для оксидных конденсаторов рабочее напряжение, которое в два раза превышает напряжение промышленной сети. На практике применяются конденсаторы с емкостью порядка десятков и сотен микрофарад. Как мы говорили выше, рабочий конденсатор используется для получения вращающего магнитного поля. Пусковая же емкость используется для получения магнитного поля, необходимого для повышения пускового момента электродвигателя. Пусковой конденсатор подключается параллельно рабочему через центробежный выключатель. Когда есть пусковая емкость вращающееся магнитное поле асинхронного двигателя в момент пуска приближается к круговому, а магнитный поток увеличивается. Это повышает пусковой момент и улучшает характеристики двигателя. При достижении асинхронным двигателем оборотов достаточных для отключения центробежного выключателя, пусковая емкость отключается и двигатель остается в работе только с рабочим конденсатором. Схема включения рабочего и пускового конденсаторов приведены на (Рис. 1).

Схема с рабочим и пусковым конденсаторами

В таблице приведены обособленные характеристики рабочих и пусковых конденсаторов для асинхронных двигателей.

 

РАБОЧИЙ

ПУСКОВОЙ

НазначениеДля асинхронных электродвигателейДля асинхронных электродвигателей
Схема подключенияПоследовательно с пусковой обмоткой электродвигателяПараллельно рабочему конденсатору
В качествеФазосмещающего элементаФазосмещающего элемента
Для чегоДля получения кругового вращающееся магнитного поля, необходимого для работы электродвигателяДля получения магнитного поля, необходимого для повышения пускового момента электродвигателя
Время включенияВ процессе эксплуатации электродвигателяВ момент пуска электродвигателя

Эксплуатация, обслуживание и ремонт

В процессе эксплуатации насосного оборудования с однофазным асинхронным двигателем особое внимание следует обращать на питающее напряжение электрической сети. В случае пониженного напряжения сети, как известно, снижается пусковой момент и частота вращения ротора, из-за увеличения скольжения. При низком напряжении увеличивается также нагрузка на рабочий конденсатор и возрастает время запуска двигателя. В случае значительного провала напряжения питания более 15% высока вероятность того, что асинхронный двигатель не запустится. Очень часто при низком напряжении выходит из строя рабочий конденсатор из-за повышенных токов и перегрева. Он расплавляется и из него вытекает электролит. Для ремонта необходимо приобрести и установить новый конденсатор соответствующей емкости. Очень часто случается, что нужного конденсатора под рукой нет. В этом случае можно подобрать требуемую емкость из двух или даже трех и четырех конденсаторов, подключив их параллельно. Здесь следует обратить внимание на рабочее напряжение, оно должно быть не ниже, чем напряжение на заводском конденсаторе. Общая емкость конденсатора(ов) должна отличаться от номинала не более чем 5%. Если установить емкость большего номинала, то двигатель запустится в работу и будет работать, но при этом начнет греться. Если с помощью клещей измерить номинальный ток двигателя, то ток будет завышен.  Так как полное электрическое сопротивление цепи в обмотках двигателя состоит из активного сопротивления цепи и реактивного сопротивления обмоток двигателя и емкости, то с увеличением емкости общее сопротивление возрастает. Сдвиг фаз токов в обмотках из-за увеличения полного сопротивления электрической цепи обмоток после запуска двигателя сильно уменьшится, магнитное поле из синусоидального превратится в эллиптическое, и рабочие характеристики асинхронного двигателя очень сильно ухудшаются, снижается КПД и возрастают тепловые потери.

  Иногда бывает, что вместе с конденсатором выходит из строя и пусковая обмотка однофазного двигателя. В такой ситуации стоимость ремонта резко возрастает, ибо надо не только заменить конденсатор, но еще и перемотать статор. Как известно, перемотка статора одна из самых дорогих операций при ремонте двигателя. Очень редко, но бывает и такая ситуация когда при низком напряжении выходит из строя только пусковая обмотка, а конденсатор при этом остается рабочим. Для ремонта двигателя нужно перематывать статор. Все эти ситуации с двигателем случаются при низком напряжении однофазной питающей сети. Для решения этой проблемы в идеальном случае необходим стабилизатор напряжения.

Спасибо за оказанное внимание

 

P.S. Понравился пост? Порекомендуйте его своим друзьям и знакомым в социальных сетях.

Еще похожие посты по данной теме:

Разделительный конденсатор

Создание связи по переменному току необходимо, чтобы запретить протекание постоянного тока между определенными точками схемы и обес­печить при этом свободное прохождение переменного тока. Электрон­ные компоненты, обеспечивающие связь по переменному току, например конденсаторы или трансформаторы, обычно устанавливаются на входе и выходе усилителя. Таким образом, заданный режим покоя (статический режим) транзистора не влияет на статические режимы предыдущего и последующего каскадов.

В схеме, приведенной на рис. 23.1. конденсатор связывает точки А и В по переменному току, aR – нагрузочный резистор. Для постоянного тока конденсатор действует как разрыв цепи, полностью блокируя протекание постоянного тока между точками А и В. По этой причине конденсатор связи называют блокировочным или разделительным конденсатором.

Удовлетворительное качество связи по переменному току достигается только в том случае, когда реактивное сопротивление Хс конденсатора на рабочей частоте много меньше сопротивления нагрузочного резистора R. Тогда на этом конденсаторе падает (и теряется) очень малая часть напряжения входного сигнала. Например, если Vвх = 100 мВ, то связь по переменному току можно считать удовлетворительной, когда выходное напряжение          Vвых = 95 мВ и на разделительном конденсаторе падает 5 мВ (5%). Требуемую емкость разделительного конденсатора определяют два фактора.

1.                   Сопротивление загрузочного резистора R. Считая, что удовлетвори­тельная связь но переменному току достигается, когда Хс = R/20, для R = 1 кОм получаем Хс = 50 Ом.

 

 

Рис. 23.1. Установка разделительного                                      Рис. 23.2.  Влияние развязывающего конденсатора.                                                                          конденсатора.

                                                           

Указаны потен­циалы точки А без развязывающего конденсатора (а) и с развязывающим конденсатором (б).

Предположим, что рабочая частота f = 300 Гц. Поскольку Хc = 1/2πfC1, то

Если сопротивление нагрузочного резистора увеличить до 100 кОм, то Хc= R/20 = 1/20·100 = 5 кОм

Таким образом, если сопротивление нагрузочного резистора увеличить в 100 раз (с 1 кОм до 100 кОм), то емкость разделительного конденсатора можно уменьшить в той же пропорции (с 10 мкФ до 0,1 мкФ).

Вообще, чем больше сопротивление нагрузочного резистора, тем мень­ше требуемая емкость разделительного конденсатора.

2. Рабочая частота. Возьмем в качестве исходного вышеприведенный пример, где удовлетворительная связь по переменному току достига­лась при С = 10 мкФ и R = 1 кОм для f = 300 Гц.

Если теперь рабочую частоту увеличить до 300 кГц, то с учетом усло­вия Хс = R/20 = 50Ом получаем

Таким образом, если рабочую частоту увеличить в 1000 раз (с 300 Гц до 300 кГц), то емкость разделительного конденсатора можно уменьшить в 1000 раз (с 10 мкФ до 0,01 мкФ).

Вообще, при заданном сопротивлении нагрузочного резистора для низ­ких рабочих частот необходимо использовать разделительные конденсаторы большой емкости, и наоборот. связывает выход усилителя с нагрузкой или следующим каскадом, его емкость сравнима с емкостью конденсатора Ci. Типичные значения емкостей разделитель-ьшх конденсаторов следующие:

10-50 мкФ. 0.01-0,1 мкФ.

для звуковых частот:

для радиочастот:

 

Рис. 23.3. Усилитель с RC-связью с

развязывающим конденсатором С3 в цепи эмиттера.            Рис. 23.4. Инвертирование (измене­ние на 180°) фазы сигнала в усили­теле с ОЭ.

Развязывающий конденсатор

Отрицательная обратная связь через резистор R4 в усилителе на рис. 23.3, с одной стороны, обеспечивает необходимую стабильность усилителя по постоянному току, а с другой стороны, снижает его коэффициент усиле­ния до очень малой величины (2-3). Снижение коэффициента усиления связано с действием отрицательной обратной связи по переменному току, обусловленной падением напряжения сигнала на резисторе R4. Для устранения этой отрицательной обратной связи по переменному току и одновременного сохранения стабильности по постоянному току применя­ется эмиттерный развязывающий конденсатор С3.

Типичные значения емкости эмиттерного развязывающего конденса­тора того же порядка, что и для разделительного конденсатора.

Усиление

Схема, приведенная на рис. 23.3, является законченной схемой однокас­кадного усилителя с ОЭ. При подаче сигнала (например, синусоидальной формы) на вход усилителя этот сигнал передается через конденсатор С1 на базу транзистора. В начале положительного полупериода входного сигнала потенциал базы возрастает относительно потенциала эмиттера, напряжение VBEувеличивается, ток эмиттера Ie, а с ним и ток коллек­тора Ic, возрастают, в результате уменьшается напряжение на коллекторе Vc. Это означает, что положительному полу периоду входного сигнала со­ответствует отрицательный полупериод выходного сигнала. С другой сто­роны, отрицательному полупериоду входного сигнала соответствует поло­жительный полупериод изменения коллекторного напряжения. Таким образом, сигналы на входе и выходе усилителя противофазны, как по­казано на рис. 23.4. Усиление сигнала происходит в силу того, что очень малый размах напряжения VBEприводит к большому размаху тока транзистора, который, проходя через резисторR3, вызывает большой размах коллекторного напряжения.

Линия нагрузки

Выходные характеристики транзистора дают общее представление о рабо­те транзистора. Для того чтобы получить представление о работе транзи­стора в конкретной схеме, нужно начертить линию нагрузки. На рис. 23.5 изображены семейство выходных характеристик транзистора, работаю­щего в схеме усилителя на рис. 23.3, и линия нагрузки XY.

Прежде чем проводить линию нагрузки, нужно сначала зафиксиро­вать две точки, попадающие на эту линию. Лучше всего использовать точку Х на оси х, где ток Ic = 0, и точку Y на оси у, где Vc = 0. Через эти две точки проводится прямая линия — линия нагрузки. Предполагается, что Vc = VCE.

Точка X. В этой точке ток транзистора Ic = 0. Транзистор находится в состоянии отсечки. Следовательно, напряжение на коллекторе Vc = VCC.

Точка Y. Здесь коллекторное напряжение Vc = 0. Подставляя Vc = 0 в уравнение               VCC = Vc + VR3, получаем VCC = VR3. Но VR3 = Ic R3, поэтому VCC = Ic R3. Следовательно,

Ic = VCC / R3.

Рис. 23.5. Линия нагрузки.

Для величин, указанных на рис. 23.3, положение точек Х и Y будет определяться следующими параметрами:

Точка Х          Ic = 0, Vc = VCC = 10 В.

Точка Y         Vc = 0, Ic = VCC/ R3 = 10/3,3 = 3 мА.

Таким образом, XY — это линия нагрузки для нагрузочного резистора сопротивлением        R3 = 3,3 кОм.

При использовании нагрузочного резистора меньшего номинала (2,2 кОм) получаем линию нагрузки ХYa. Положение точки Х не изменяется по сравнению с предыдущим случаем, поскольку напряжение VСС остается тем же самым — 10 В. Для точки Yb получаем Ic = VCC / R3 = 10 В/2,2кОм = 4,55мА.

Нагрузочному резистору более высокого номинала, например 4,9 кОм, соответствует линия нагрузки ХYb с точкой Yb при Ic = 10 В/4, 9 кОм ≈ 2 мА.

Графический анализ

Процесс усиления сигнала осуществляется вдоль линии нагрузки и может быть представлен графически, как показано на рис. 23.6. Точка Q есть статическая рабочая точка, представляющая режим работы усилителя по постоянному току, т. е. в отсутствие сигнала. Рабочая точка задает смещение транзистора в статическом режиме. В рассматриваемом случае смещение определяется следующими величинами:

Ib = 20 мкА, Ic = 1,5 мА, Vc = 5 В.

Рис. 23.6. Графическое представление работы усилителя.

 

Рис. 23.7. Перегрузка усилителя, приводящая к ограничению выходного сиг­нала.

При подаче сигнала базовый ток изменяется по синусоиде с амплитудой 20 мкА (от 0 до 40 мкА). Это приводит к изменению коллекторного тока Ic с размахом 2,8 мА и изменению коллекторного напряжения с размахом около 9 В.

С одной стороны размах входного сигнала ограничен линией Ib = 0, соответствующей отсечке транзистора (точка М на линии нагрузки), а с другой стороны – линией Ib = 40 мкА, соответствующей насыщению транзистора (точка N на линии нагрузки). Для рассматриваемого уси­лителя рабочая точка Q выбирается в середине линии нагрузки. В этом случае при подаче сигнала с амплитудой 20 мкА на базу транзистора базовый ток изменяется в пределах от 0 до 40 мкА, обеспечивая максимальную величину неискаженного выходного сигнала.

 

Рис. 23.8. Графическое представление работы усилителя с использованием пе­редаточной характеристики.

Любая попыт­ка превышения этой величины входного сигнала приводит к искажению формы выходного сигнала. Это хорошо видно на рис. 23.7, где иллюстри­руется случай перегрузки усилителя с результирующим ограничением синусоидального сигнала. Входной и выходной сигналы могут быть так­же представлены графически с помощью передаточной характеристики транзистора (рис. 23.8). Рабочий диапазон усилителя ограничен линей­ным участком характеристики передачи, выход за границы этого участка приводит к искажениям.

Добавить комментарий

Молекулярные выражения: электричество и магнетизм

Факторы, влияющие на емкость

Конденсатор — это электрическое устройство, предназначенное для накопления электрического заряда, обычно состоящее из двух параллельных проводящих пластин, разделенных изолирующим слоем, называемым диэлектриком.

Щелкайте стрелки, чтобы выбрать различные комбинации диэлектриков, площадей пластин и расстояний.

На емкость конденсатора влияет площадь пластин, расстояние между пластинами и способность диэлектрика выдерживать электростатические силы.В этом руководстве показано, как изменение этих параметров влияет на емкость конденсатора. Пластины большего размера обеспечивают большую емкость для хранения электрического заряда. Следовательно, с увеличением площади пластин увеличивается емкость.

Емкость прямо пропорциональна электростатическому силовому полю между пластинами. Это поле тем сильнее, когда пластины расположены ближе друг к другу. Следовательно, с уменьшением расстояния между пластинами емкость увеличивается.

Диэлектрические материалы оцениваются на основе их способности выдерживать электростатические силы с точки зрения числа, называемого диэлектрической проницаемостью.Чем выше диэлектрическая проницаемость, тем выше способность диэлектрика выдерживать электростатические силы. Следовательно, с увеличением диэлектрической проницаемости увеличивается емкость.

НАЗАД К РУКОВОДСТВАМ ПО ЭЛЕКТРИЧЕСТВЕ И МАГНЕТИЗМУ

Вопросы или комментарии? Отправить нам письмо.
© 1995-2019, автор — Майкл В. Дэвидсон и Государственный университет Флориды. Все права защищены. Никакие изображения, графика, программное обеспечение, сценарии или апплеты не могут быть воспроизведены или использованы каким-либо образом без разрешения правообладателей.Использование этого веб-сайта означает, что вы соглашаетесь со всеми юридическими положениями и условиями, изложенными владельцами.
Этот веб-сайт поддерживается нашим

Команда разработчиков графики и веб-программирования
в сотрудничестве с оптической микроскопией в Национальной лаборатории сильного магнитного поля
.
Последнее изменение: среда, 7 июня 2017 г., 12:21
Счетчик доступа с 3 апреля 1999 г .: 435249

факторов, влияющих на емкость | Конденсаторы

Существует три основных фактора конструкции конденсатора, определяющих величину создаваемой емкости.Все эти факторы определяют емкость, влияя на то, какой поток электрического поля (относительная разница электронов между пластинами) будет развиваться для данной величины силы электрического поля (напряжения между двумя пластинами):

ПЛОЩАДЬ ПЛАСТИНЫ : При прочих равных условиях большая площадь пластины дает большую емкость; меньшая площадь пластины дает меньшую емкость.

Пояснение: Чем больше площадь пластины, тем больше магнитный поток (заряд, собранный на пластинах) для данной силы поля (напряжение на пластинах).

РАССТОЯНИЕ ПЛАСТИН : При прочих равных условиях большее расстояние между пластинами дает меньшую емкость; меньшее расстояние между пластинами дает большую емкость.

Пояснение: Более близкое расстояние приводит к большей силе поля (напряжение на конденсаторе, деленное на расстояние между пластинами), что приводит к большему потоку поля (заряд, накопленный на пластинах) для любого заданного напряжения, приложенного к пластинам.

ДИЭЛЕКТРИЧЕСКИЙ МАТЕРИАЛ : При прочих равных условиях большая диэлектрическая проницаемость диэлектрика дает большую емкость; меньшая диэлектрическая проницаемость диэлектрика дает меньшую емкость.

Пояснение: Хотя это сложно объяснить, некоторые материалы предлагают меньшее сопротивление потоку поля для данной величины силы поля. Материалы с большей диэлектрической проницаемостью допускают больший поток поля (предлагают меньшее сопротивление) и, следовательно, больший накопленный заряд для любой заданной величины силы поля (приложенного напряжения).

«Относительная» диэлектрическая проницаемость означает диэлектрическую проницаемость материала относительно диэлектрической проницаемости чистого вакуума. Чем больше число, тем больше диэлектрическая проницаемость материала.Стекло, например, с относительной диэлектрической проницаемостью 7, имеет в семь раз большую диэлектрическую проницаемость, чем чистый вакуум, и, следовательно, позволяет создать поток электрического поля, в семь раз более сильный, чем у вакуума, при прочих равных условиях. В следующей таблице перечислены относительные диэлектрические проницаемости (также известные как «диэлектрическая проницаемость») различных распространенных веществ:

Материал

Относительная диэлектрическая проницаемость (диэлектрическая проницаемость)
Вакуум 1.0000
Воздух 1.0006
PTFE, FEP («тефлон») 2,0
Полипропилен от 2,20 до 2,28
Смола АБС от 2,4 до 3,2
Полистирол от 2,45 до 4,0
Вощеная бумага 2,5
Масло трансформаторное от 2,5 до 4
Твердая резина 2.От 5 до 4,80
Дерево (Дуб) 3,3
Силиконы от 3,4 до 4,3
Бакелит от 3,5 до 6,0
Кварц плавленый 3,8
Дерево (клен) 4,4
Стекло от 4,9 до 7,5
Касторовое масло 5,0
Дерево (береза) 5,2
Слюда, мусковит 5.От 0 до 8,7
Стекловолоконная слюда от 6,3 до 9,3
Фарфор, стеатит 6,5
Глинозем от 8,0 до 10,0
Вода дистиллированная 80,0
Барий-стронций-титанит 7500

Примерную емкость для любой пары разделенных проводов можно найти по следующей формуле:

Конденсатор можно сделать переменным, а не фиксированным, путем изменения любого из физических факторов, определяющих емкость.Один относительно простой фактор, который можно изменить в конструкции конденсатора, — это площадь пластины или, точнее, величина перекрытия пластин.

На следующей фотографии показан пример переменного конденсатора, использующего набор чередующихся металлических пластин и воздушный зазор в качестве диэлектрического материала:

При вращении вала степень перекрытия наборов пластин будет изменяться, изменяя эффективную площадь пластин, между которыми может быть установлено концентрированное электрическое поле.Этот конкретный конденсатор имеет емкость в пикофарадном диапазоне и находит применение в радиосхемах.

СВЯЗАННЫЕ РАБОЧИЕ ЛИСТЫ:

Емкость | Основы | Направляющая конденсатора

Что такое емкость?

Электрическая емкость — это способность проводящего тела накапливать заряд. Значение емкости конденсатора определяется по формуле:

, где C — емкость, Q — количество заряда, накопленного на каждом электроде, а V — напряжение между двумя электродами.В реальных схемах количество заряда на одной пластине равно количеству заряда на другой пластине конденсатора, но эти два заряда имеют разные знаки. Изучив эту формулу, можно сделать вывод, что конденсатор емкостью 1 Ф удерживает заряд 1 Кл, когда на его два вывода подается напряжение 1 В.

Единица емкости

Единица измерения емкости — Фарад [Ф]. Этот блок может быть несколько непрактичным. С точки зрения большинства инженеров-электриков, одна фарада — это огромное значение емкости.В большинстве электронных схем используются конденсаторы емкостью до нескольких мФ. Для этого есть несколько веских причин.

Одна из причин заключается в том, что при работе с сигналами в электрической цепи, когда частота сигнала увеличивается, потребность в конденсаторах большой емкости уменьшается, потому что на более высоких частотах даже небольшой конденсатор может оказать большое влияние на схему. Поскольку большинство современных цифровых схем имеет тенденцию переходить на более высокие частоты, чтобы удовлетворить требования по повышению скорости обработки, в этих схемах в основном используются конденсаторы емкостью до нескольких мФ.В результате практически отсутствует потребность в конденсаторах большой емкости в частях электрических цепей, обрабатывающих сигналы.

Другая причина заключается в том, что конденсаторы с высокой емкостью имеют большие физические размеры. Поэтому использование таких конденсаторов избегается, особенно в мобильных устройствах. Однако в последнее время в области суперконденсаторов произошел технологический прогресс. Благодаря этим достижениям теперь можно производить конденсаторы с емкостью до нескольких тысяч фарад, которые по размеру сопоставимы с банкой для напитков.Эти суперконденсаторы находятся в стадии разработки, чтобы заменить батареи в различных приложениях, таких как электромобили или даже сотовые телефоны, если ученым удастся уменьшить их размер до приемлемой степени. Суперконденсатор 1F меньшего размера может быть изготовлен по размеру, сопоставимому с размером нескольких десятицентовиков, уложенных друг на друга.

Емкостные характеристики реальных конденсаторов

Идеальный конденсатор имеет фиксированное значение емкости. Однако емкость реального конденсатора может измениться по нескольким причинам.В большинстве случаев диэлектрик, используемый в конденсаторе, не идеален, и на диэлектрическую проницаемость могут влиять определенные факторы.

Напряжение , приложенное к конденсатору, может изменить диэлектрическую проницаемость диэлектрика. Это изменение напрямую влияет на емкость такого конденсатора. Например, этот эффект ярко выражен, когда в качестве диэлектрика используются сегнетоэлектрические материалы. Такой эффект может быть полезен в некоторых случаях, и существуют специальные устройства, такие как варикап-диод, которые используют зависящую от напряжения емкость в цепи.Название varicap происходит от слов VARIable CAPacitance. Эти диоды используются в качестве конденсаторов с регулируемым напряжением и иногда используются в радио- и ТВ-тюнерах, контурах фазовой автоподстройки частоты и усилителях, а также в других схемах.

Частота сигнала, присутствующего на выводах конденсатора, также может влиять на его емкость. Этот эффект называется диэлектрической дисперсией и возникает из-за того, что поляризация диэлектрика отстает от быстро меняющегося сигнала. На низких частотах сигнала этот эффект незначителен, однако на высоких частотах этот эффект может быть весьма заметным.Примером применения, в котором необходимо учитывать частотно-зависимые изменения емкости, являются МОП-транзисторы, емкость затвора которых зависит от частоты.

Возраст конденсатора также влияет на его емкость. Некоторые конденсаторы более стабильны во времени, в то время как другие имеют относительно короткий срок службы из-за эффектов старения. Например, электролит в электролитическом конденсаторе может высохнуть в течение многих лет, даже если он не используется в цепи. По прошествии времени емкость изменяется от расчетного значения, и такое изменение может в конечном итоге привести к неисправности цепи.По этой причине некоторые конденсаторы имеют определенный расчетный срок хранения, а также расчетный срок службы в цепи.

Паразитная емкость

В то время как конденсаторы предназначены для преднамеренного введения определенной емкости в цепь, в некоторых других устройствах, таких как резисторы, катушки индуктивности или даже обычные провода, емкость считается нежелательным паразитным эффектом. Это особенно важно на высоких частотах, где такие емкости могут сильно повлиять на работу цепи.Если две сигнальные линии в интегральных схемах проходят рядом друг с другом, паразитная емкость может вносить значительный шум в одну линию, если другая линия управляется высокочастотным сигналом, например, тактовой частотой 3,2 ГГц. Это называется перекрестными помехами и может создать серьезную проблему для инженеров по интегральным схемам. Чем длиннее две линии, проходящие рядом друг с другом и чем меньше расстояние между ними, тем сильнее выражен этот эффект. Чтобы противодействовать этой проблеме, разработчик схемы должен поддерживать достаточное расстояние между такими линиями.В чувствительных высокочастотных цепях, таких как генераторы и фильтры, в процессе проектирования необходимо учитывать даже небольшие паразитные емкости таких простых устройств, как резисторы. Паразитные емкости никогда нельзя полностью устранить, но их можно уменьшить, используя короткие выводы, предпочтительно технологию SMD и устройства, специально разработанные для уменьшения их емкости, такие как резисторы с малой емкостью. Например, если микроконтроллер использует кварцевый генератор, практическое правило состоит в том, чтобы разместить кристалл и все поддерживающие элементы физически как можно ближе к микроконтроллеру, чтобы уменьшить емкость, генерируемую проводами на печатной плате (печатной плате) и тем самым шум, возникающий из-за перекрестных помех.

факторов, влияющих на емкость | Диэлектрическая проницаемость

На емкость конденсаторов влияют три основных фактора, которые будут подробно обсуждаться в этом руководстве.

Единица измерения емкости в системе СИ — фарад, названная в честь английского физика и химика Майкла Фарадея. Единица измерения фарада — F. Емкость — это способность компонента удерживать заряд. Количество накопленного заряда Q пропорционально приложенному напряжению. Заряд также определяется емкостью конденсатора.Связь этих терминов может быть выражена следующим образом:

$ Q = CV $

Где

Q = электрический заряд, измеряемый в кулонах

C = емкость в фарадах

V = электрический потенциал на конденсаторе в вольтах

В терминах постоянного тока это уравнение можно сформулировать следующим образом:

Конденсатор имеет емкость в одну фараду, если один вольт, приложенный к пластинам, производит заряд в один кулон в конденсаторе.

Пример

A 0.{-6}} \ text {F} $

Единица измерения емкости также может быть описана в терминах переменного тока (который постоянно изменяется). Если V изменяется со скоростью один вольт в секунду и вызывает протекание одного кулона заряда в секунду, емкость равна одной фараде. Поскольку 1 Кл / с равен 1 А, мы можем сказать, что емкость устройства равна одной фараде, если через цепь протекает один ампер, когда приложенная ЭДС изменяется со скоростью один вольт в секунду.

Фарад — это чрезвычайно большая единица измерения емкости.На практике удобнее использовать небольшую единицу фарад, такую ​​как микрофарад (мкФ), которая составляет 1 * 10 -6 . Фактически, для очень высокочастотных приложений чаще используются нанофарады (нФ) и пикофарады (пФ). Эти единицы равны 1 * 10 -9 F и 1 * 10 -12 F соответственно.

Пример

Определенный конденсатор рассчитан на 0,0068 F. Какова его емкость в микрофарадах и нанофарадах?

Раствор

Один фарад равен 1 * 10 6 мкФ.{\ text {3}}} = 6 800 000 \ text {nF} $

Диэлектрическая проницаемость (k)

Отношение емкости конденсатора с данным диэлектриком к емкости идентичного конденсатора, содержащего воздух или вакуум для его диэлектрика.

Одним из основных факторов, влияющих на емкость конденсатора, является тип диэлектрического материала, используемого между пластинами. Эти материалы, изоляторы, оцениваются по их способности создавать диэлектрический поток с точки зрения параметра, называемого диэлектрической проницаемостью (k).Материалы с высокой диэлектрической проницаемостью могут создавать большую емкость, чем материалы с низким k, при той же площади пластины и расстоянии.

Стандарт необходим для сравнения диэлектрической проницаемости одного материала с другим. Этим стандартом служит вакуум, которому присваивается значение единицы. По сути, такую ​​же ценность имеет воздух. Диэлектрическая проницаемость всех материалов сравнивается с диэлектрической проницаемостью вакуума. Таким образом, если конкретный диэлектрический материал имеет значение k, равное 6,5, он будет производить в 6,5 раз большую емкость, чем если бы в конденсаторе такого же размера использовался вакуум.Диэлектрическая прочность ряда материалов приведена в таблице 1.

Материал Диэлектрическая проницаемость k
Воздух или вакуум 1
Бумага 2.0-6.0
Пластик 2,1-6,0
Минеральное масло 2,2-2,3
Полистирол 2,6
Кремниевое масло 2,7-2.8
Кварц 3,8-4,4
Стекло 4,8-8,0
Фарфор 5,1-5,9
Слюда 5,4-8,7
Askarel Oil 5,6 -5,9
Оксид алюминия 8,4
Пятиокись тантала 26
Керамика 12-400000

Таблица.1: Диэлектрическая проницаемость

Факторы, влияющие на емкость

Несколько факторов влияют на способность конденсатора накапливать электрический заряд. Они следующие:

  1. Площадь пластин
  2. Расстояние между пластинами
  3. Диэлектрическая проницаемость материала между пластинами

Связь этих параметров можно понять, если обратиться к элементному конденсатору с параллельными пластинами. показано на рисунке 1.

Рис.1: Конденсатор с параллельными пластинами

Следующая формула также выражает взаимосвязь:

$ C = 0,224 \ frac {kA} {d} $

Где

K = диэлектрическая проницаемость материала между пластины

A = площадь одной пластины в квадратных дюймах

D = расстояние между пластинами в дюймах

0,224 = коэффициент преобразования при использовании дюймов

Если размеры указаны в сантиметрах (см), используйте следующую формулу:

$ С = 0.08842 \ frac {kA} {d} $

Где

K = диэлектрическая проницаемость материала между пластинами

A = площадь одной пластины в квадратных см

D = расстояние между пластинами в см

0,224 = коэффициент пересчета при использовании сантиметров

Анализ этих формул показывает, что емкость напрямую зависит от площади пластины и обратно пропорциональна расстоянию между пластинами. Диэлектрическая проницаемость также напрямую влияет на емкость. Например, если в качестве диэлектрика использовать слюду, емкость увеличится с 5.От 4 до 8,7 раз.

Если конденсатор состоит из более чем двух параллельных пластин, его емкость в пикофарадах рассчитывается по следующей формуле:

$ C = 0,08842 \ frac {kA} {d} (n-1) $

Где n — количество тарелок.

Пример

Простой конденсатор имеет две параллельные пластины, разделенные на 2 см. если площадь одной пластины составляет 100 см 2 , а диалектика — полистирол, какова емкость в пикофарадах?

Решение

$ C = 0.08842 * \ frac {2.6 * 100} {2} = 11.49 \ text {pF} $

В приведенном выше решении 2.6 — диэлектрическая проницаемость полистирола, указанная в таблице 1.

Конденсаторы — learn.sparkfun.com

Добавлено в избранное Любимый 73

Теория конденсаторов

Примечание : Материал на этой странице не совсем критичен для понимания новичками в электронике … и к концу все становится немного сложнее. Мы рекомендуем прочитать раздел Как делается конденсатор , остальные, вероятно, можно было бы пропустить, если они вызывают у вас головную боль.

Как делается конденсатор

Условное обозначение конденсатора на самом деле очень похоже на то, как он сделан. Конденсатор состоит из двух металлических пластин и изоляционного материала, называемого диэлектриком . Металлические пластины расположены очень близко друг к другу, параллельно, но между ними находится диэлектрик, чтобы они не соприкасались.

Ваш стандартный конденсаторный сэндвич: две металлические пластины, разделенные изолирующим диэлектриком.

Диэлектрик может быть изготовлен из всех видов изоляционных материалов: бумаги, стекла, резины, керамики, пластика или всего, что препятствует прохождению тока.

Пластины изготовлены из проводящего материала: алюминия, тантала, серебра или других металлов. Каждый из них подключен к клеммному проводу, который в конечном итоге подключается к остальной части схемы.

Емкость конденсатора — сколько в нем фарад — зависит от его конструкции. Для большей емкости требуется конденсатор большего размера. Пластины с большей площадью перекрытия поверхности обеспечивают большую емкость, в то время как большее расстояние между пластинами означает меньшую емкость. Материал диэлектрика даже влияет на то, сколько фарад имеет колпачок.Полная емкость конденсатора может быть рассчитана по формуле:

где ε r — относительная диэлектрическая проницаемость диэлектрика (постоянное значение, определяемое материалом диэлектрика), A — площадь перекрытия пластин друг с другом, а d — расстояние между пластинами.

Как работает конденсатор

Электрический ток — это поток электрического заряда, который электрические компоненты используют, чтобы загораться, вращаться или делать то, что они делают.Когда ток течет в конденсатор, заряды «застревают» на пластинах, потому что они не могут пройти через изолирующий диэлектрик. Электроны — отрицательно заряженные частицы — засасываются одной из пластин, и она становится в целом отрицательно заряженной. Большая масса отрицательных зарядов на одной пластине отталкивает, как заряды, на другой пластине, делая ее заряженной положительно.

Положительный и отрицательный заряды на каждой из этих пластин притягиваются друг к другу, потому что это то, что делают противоположные заряды.Но с диэлектриком, сидящим между ними, как бы они ни хотели соединиться, заряды навсегда останутся на пластине (до тех пор, пока им не будет куда-то идти). Неподвижные заряды на этих пластинах создают электрическое поле, которое влияет на электрическую потенциальную энергию и напряжение. Когда заряды группируются на таком конденсаторе, крышка накапливает электрическую энергию так же, как батарея может накапливать химическую энергию.

Зарядка и разрядка

Когда на пластинах конденсатора сливаются положительный и отрицательный заряды, конденсатор становится на заряженным .Конденсатор может сохранять свое электрическое поле — удерживать свой заряд, потому что положительный и отрицательный заряды на каждой из пластин притягиваются друг к другу, но никогда не достигают друг друга.

В какой-то момент пластины конденсатора будут настолько заряжены, что просто не смогут больше принимать их. На одной пластине достаточно отрицательных зарядов, чтобы они могли отразить любые другие, которые попытаются присоединиться. Здесь вступает в игру емкость конденсатора , (фарады), которая говорит вам о максимальном количестве заряда, которое может хранить конденсатор.

Если в цепи создается путь, который позволяет зарядам найти другой путь друг к другу, они выйдут из конденсатора, и он разрядит .

Например, в схеме ниже можно использовать аккумулятор для создания электрического потенциала на конденсаторе. Это вызовет нарастание одинаковых, но противоположных зарядов на каждой из пластин, пока они не станут настолько полными, что оттолкнут ток от протекания. Светодиод, расположенный последовательно с крышкой, может обеспечивать путь для тока, а энергия, запасенная в конденсаторе, может использоваться для кратковременного освещения светодиода.

Расчет заряда, напряжения и тока

Емкость конденсатора — сколько в нем фарад — говорит вам, сколько заряда он может хранить. Сколько заряда в настоящее время хранит конденсатор , зависит от разности потенциалов (напряжения) между его пластинами. Это соотношение между зарядом, емкостью и напряжением можно смоделировать с помощью следующего уравнения:

Заряд (Q), накопленный в конденсаторе, является произведением его емкости (C) и приложенного к нему напряжения (V).

Емкость конденсатора всегда должна быть постоянной известной величиной. Таким образом, мы можем регулировать напряжение, чтобы увеличивать или уменьшать заряд крышки. Больше напряжения означает больше заряда, меньше напряжения … меньше заряда.

Это уравнение также дает нам хороший способ определить значение одного фарада. Один фарад (F) — это способность хранить одну единицу энергии (кулоны) на каждый вольт.

Расчет тока

Мы можем пойти дальше по уравнению заряда / напряжения / емкости, чтобы выяснить, как емкость и напряжение влияют на ток, потому что ток — это скорость потока заряда.Суть отношения конденсатора к напряжению и току такова: величина тока , проходящего через конденсатор , зависит как от емкости, так и от того, как быстро напряжение растет или падает, . Если напряжение на конденсаторе быстро растет, через конденсатор будет индуцироваться большой положительный ток. Более медленный рост напряжения на конденсаторе означает меньший ток через него. Если напряжение на конденсаторе стабильное и неизменное, через него не будет проходить ток.

(Это некрасиво, и это касается вычислений. Это не все, что нужно, пока вы не перейдете к анализу во временной области, разработке фильтров и другим грубым вещам, поэтому переходите к следующей странице, если вам не нравится это уравнение .) Уравнение для расчета тока через конденсатор:

Часть dV / dt этого уравнения является производной (причудливый способ сказать мгновенной скорости ) напряжения во времени, это эквивалентно тому, как «насколько быстро напряжение растет или падает в этот самый момент».Большой вывод из этого уравнения заключается в том, что если напряжение стабильно, , производная равна нулю, что означает, что ток также равен нулю . Вот почему ток не может течь через конденсатор, поддерживающий постоянное постоянное напряжение.



← Предыдущая страница
Условные обозначения и единицы

Почему расстояние между пластинами конденсатора влияет на его емкость?

Чтобы получить техническую информацию, вы хотите взглянуть на закон Кулона.2} \ $

Что сообщает нам силу на расстоянии \ $ r \ $ от точечного заряда \ $ q \ $.

Если вы хотите получить действительно технических, тогда вам нужно начать читать о квантовой механике и взаимодействиях между частицами и задействованных в ней энергиях.

Когда две частицы (скажем, в данном случае электроны) взаимодействуют, они посылают между собой квантовые частицы (фотоны). Им, как крысам в подвале, требуется энергия, чтобы двигаться. Чем больше расстояние, тем выше энергия.Чем выше энергия, необходимая для перемещения фотонов, тем меньше заряда остается между двумя пластинами.

Это очень упрощенный взгляд на это, и есть еще одна чертовски большая деталь, которую предстоит открыть — такие вещи, как квантовое туннелирование, лептоны, фермионы, бозоны и т. Д. Это увлекательное чтение, если у вас есть время. Я бы рекомендовал книгу Стивена Хокинга «Краткая история времени » в качестве хорошей отправной точки. Следуйте за этим с помощью суперструн Ф. Дэвида Пита и «Поиск теории всего», и вы не ошибетесь.Хотя обе эти книги сейчас становятся немного длиннее, а теории все еще развиваются, они дают хорошее представление о том, как устроена Вселенная на субатомном уровне.

Конденсаторы и диэлектрики | Физика II

Цели обучения

К концу этого раздела вы сможете:

  • Опишите действие конденсатора и определите емкость.
  • Объясните, почему конденсаторы с параллельными пластинами и их емкости.
  • Обсудите процесс увеличения емкости диэлектрика.
  • Определите емкость при заданном заряде и напряжении.

Конденсатор — это устройство, используемое для хранения электрического заряда. Конденсаторы имеют разные применения: от фильтрации статического электричества при радиоприеме до накопления энергии в дефибрилляторах сердца. Обычно в промышленных конденсаторах две проводящие части расположены близко друг к другу, но не соприкасаются, как показано на рисунке 1. (В большинстве случаев между двумя пластинами используется изолятор для обеспечения разделения — см. Обсуждение диэлектриков ниже.) Когда клеммы аккумулятора подключены к изначально незаряженному конденсатору, равные количества положительного и отрицательного заряда, + Q и — Q , разделяются на его две пластины. Конденсатор в целом остается нейтральным, но в этом случае мы называем его хранящим заряд Q .

Рис. 1. Оба конденсатора, показанные здесь, были изначально разряжены перед подключением к батарее. Теперь у них разделены заряды + Q и — Q на своих двух половинах.(а) Конденсатор с параллельными пластинами. (b) Скрученный конденсатор с изоляционным материалом между двумя проводящими листами.

Конденсатор

Конденсатор — это устройство, используемое для хранения электрического заряда.

Количество заряда Q , которое может хранить конденсатор , зависит от двух основных факторов — приложенного напряжения и физических характеристик конденсатора, таких как его размер.

Количество заряда

Q , которое может хранить конденсатор

Количество заряда Q , которое может хранить конденсатор , зависит от двух основных факторов — приложенного напряжения и физических характеристик конденсатора, таких как его размер.

Рис. 2. Линии электрического поля в этом конденсаторе с параллельными пластинами, как всегда, начинаются с положительных зарядов и заканчиваются отрицательными. Поскольку напряженность электрического поля пропорциональна плотности силовых линий, она также пропорциональна количеству заряда на конденсаторе.

Система, состоящая из двух идентичных параллельных проводящих пластин, разделенных расстоянием, как на рисунке 2, называется конденсатором с параллельными пластинами . Легко увидеть взаимосвязь между напряжением и накопленным зарядом для конденсатора с параллельными пластинами, как показано на рисунке 2.Каждая линия электрического поля начинается с отдельного положительного заряда и заканчивается отрицательным, так что линий поля будет больше, если заряд будет больше. (Рисование одной силовой линии для каждого заряда — это только удобство. Мы можем нарисовать много силовых линий для каждого заряда, но их общее количество пропорционально количеству зарядов.) Таким образом, напряженность электрического поля прямо пропорциональна Ом. .

Поле пропорционально начислению:

E Q ,

, где символ ∝ означает «пропорционально.Из обсуждения в статье «Электрический потенциал в однородном электрическом поле» мы знаем, что напряжение на параллельных пластинах равно

.

V = Ed .

Таким образом, V E . Отсюда следует, что V Q , и, наоборот,

Q V .

В целом это верно: чем больше напряжение, приложенное к любому конденсатору, тем больше в нем хранится заряд.

Различные конденсаторы будут накапливать разное количество заряда для одного и того же приложенного напряжения, в зависимости от их физических характеристик.Мы определяем их емкость C так, чтобы заряд Q , хранящийся в конденсаторе, был пропорционален C . Заряд, накопленный в конденсаторе, равен

.

Q = CV .

Это уравнение выражает два основных фактора, влияющих на количество накопленного заряда. Этими факторами являются физические характеристики конденсатора C и напряжение В . Изменив уравнение, мы видим, что емкость C — это количество заряда, накопленного на вольт, или

.

[латекс] C = \ frac {Q} {V} \\ [/ latex].

Емкость

Емкость C — величина накопленного заряда на вольт, или

[латекс] C = \ frac {Q} {V} \\ [/ latex]

Единица измерения емкости — фарад (Ф), названная в честь Майкла Фарадея (1791–1867), английского ученого, внесшего вклад в области электромагнетизма и электрохимии. Поскольку емкость — это заряд на единицу напряжения, мы видим, что фарад — это кулон на вольт, или

.

[латекс] 1 \ text {F} = \ frac {1 \ text {C}} {1 \ text {V}} \\ [/ latex].

Конденсатор емкостью 1 фарад может хранить 1 кулон (очень большое количество заряда) при подаче всего 1 вольт. Таким образом, одна фарада — это очень большая емкость. Типичные конденсаторы варьируются от долей пикофарада (1 пФ = 10 −12 Ф) до миллифарадов (1 мФ = 10 −3 Ф).

На рисунке 3 показаны некоторые распространенные конденсаторы. Конденсаторы в основном изготавливаются из керамики, стекла или пластика, в зависимости от назначения и размера. Как обсуждается ниже, в их конструкции обычно используются изоляционные материалы, называемые диэлектриками.

Рисунок 3. Некоторые типичные конденсаторы. Размер и значение емкости не обязательно связаны. (Источник: Windell Oskay)

Конденсатор с параллельными пластинами

Рис. 4. Конденсатор с параллельными пластинами, разделенные обкладками на расстояние d. Каждая пластина имеет площадь A.

Конденсатор с параллельными пластинами, показанный на рисунке 4, имеет две идентичные проводящие пластины, каждая из которых имеет площадь поверхности A , разделенных расстоянием d (без материала между пластинами).Когда на конденсатор подается напряжение В , он сохраняет заряд Q , как показано. Мы можем увидеть, как его емкость зависит от A и d , рассмотрев характеристики кулоновской силы. Мы знаем, что одинаковые заряды отталкиваются, в отличие от зарядов притягиваются, и сила между зарядами уменьшается с расстоянием. Поэтому кажется вполне разумным, что чем больше пластины, тем больше заряда они могут хранить, потому что заряды могут расходиться больше. Таким образом, C должен быть больше для большего A .Точно так же, чем ближе пластины расположены друг к другу, тем сильнее на них притяжение противоположных зарядов. Таким образом, C должно быть больше для меньшего d .

Можно показать, что для конденсатора с параллельными пластинами есть только два фактора ( A, и d ), которые влияют на его емкость C . Емкость конденсатора с параллельными пластинами в форме уравнения равна

.

[латекс] C = \ epsilon_ {o} \ frac {A} {d} \\ [/ latex].

Емкость параллельного пластинчатого конденсатора

[латекс] C = \ epsilon_ {o} \ frac {A} {d} \\ [/ latex]

A — это площадь одной пластины в квадратных метрах, а d — это расстояние между пластинами в метрах.Константа ε 0 — диэлектрическая проницаемость свободного пространства; его числовое значение в единицах СИ составляет ε 0 = 8,85 × 10 −12 Ф / м. Единицы измерения Ф / м эквивалентны C 2 / Н · м 2 . Небольшое числовое значение ε 0 связано с большим размером фарада. Конденсатор с параллельными пластинами должен иметь большую площадь, чтобы его емкость приближалась к фарадам. (Обратите внимание, что приведенное выше уравнение действительно, когда параллельные пластины разделены воздухом или свободным пространством.Когда между пластинами помещается другой материал, уравнение изменяется, как обсуждается ниже.)

Пример 1. Емкость и заряд в параллельном пластинчатом конденсаторе

  1. Какова емкость конденсатора с параллельными пластинами, каждая из которых имеет площадь 1,00 м 2 , разделенных расстоянием 1,00 мм?
  2. Какой заряд сохраняется в этом конденсаторе, если к нему приложено напряжение 3,00 × 10 3 В?
Стратегия

Определение емкости C представляет собой прямое приложение уравнения [латекс] C = \ epsilon_ {o} \ frac {A} {d} \\ [/ latex].{-9} \ text {F} = 8.85 \ text {nF} \ end {array} \\ [/ latex]

Обсуждение части 1

Это небольшое значение емкости указывает на то, насколько сложно изготовить устройство с большой емкостью. {6} \ text {V / m} \\ [/ latex]

Этого электрического поля достаточно, чтобы вызвать пробой в воздухе.

Диэлектрик

Предыдущий пример подчеркивает сложность сохранения большого количества заряда в конденсаторах. Если d сделать меньше, чтобы получить большую емкость, то максимальное напряжение должно быть уменьшено пропорционально, чтобы избежать пробоя (поскольку [латекс] E = \ frac {V} {d} \\ [/ latex]). Важным решением этой проблемы является размещение изоляционного материала, называемого диэлектриком , между пластинами конденсатора и обеспечение минимально возможного размера d .Мало того, что меньший d увеличивает емкость, многие изоляторы могут выдерживать более сильные электрические поля, чем воздух, перед тем, как сломаться.

Есть еще одно преимущество использования диэлектрика в конденсаторе. В зависимости от используемого материала емкость больше, чем та, которая определяется уравнением [латекс] C = \ kappa \ epsilon_ {0} \ frac {A} {d} \\ [/ latex], на коэффициент κ , называемый диэлектрическая проницаемость . Конденсатор с параллельными пластинами с диэлектриком между пластинами имеет емкость, определяемую выражением [латекс] C = \ kappa \ epsilon_ {0} \ frac {A} {d} \\ [/ latex] (конденсатор с параллельными пластинами с диэлектриком).

Значения диэлектрической проницаемости κ для различных материалов приведены в таблице 1. Обратите внимание, что κ для вакуума равно 1, поэтому приведенное выше уравнение справедливо и в этом случае. Если использовать диэлектрик, например, поместив тефлон между пластинами конденсатора в примере 1, то емкость будет больше в κ раз, что для тефлона составляет 2,1.

Эксперимент на вынос: создание конденсатора

Насколько большой конденсатор можно сделать из обертки от жевательной резинки? Пластины будут из алюминиевой фольги, а разделитель (диэлектрик) между ними — из бумаги.

Таблица 1. Диэлектрическая проницаемость и диэлектрическая прочность для различных материалов при 20ºC
Материал Диэлектрическая проницаемость κ Диэлектрическая прочность (В / м)
Вакуум 1,00000
Воздух 1.00059 3 × 10 6
Бакелит 4,9 24 × 10 6
Плавленый кварц 3.78 8 × 10 6
Неопреновый каучук 6,7 12 × 10 6
Нейлон 3,4 14 × 10 6
Бумага 3,7 16 × 10 6
Полистирол 2,56 24 × 10 6
Стекло Pyrex 5,6 14 × 10 6
Кремниевое масло 2.5 15 × 10 6
Титанат стронция 233 8 × 10 6
тефлон 2,1 60 × 10 6
Вода 80

Обратите внимание, что диэлектрическая проницаемость воздуха очень близка к 1, так что конденсаторы, заполненные воздухом, действуют так же, как конденсаторы с вакуумом между пластинами , за исключением , что воздух может стать проводящим, если напряженность электрического поля станет равной. слишком большой.(Напомним, что [латекс] E = \ frac {V} {d} \\ [/ latex] для конденсатора с параллельными пластинами.) В таблице 1 также показаны максимальные напряженности электрического поля в В / м, называемые диэлектрической прочностью , для нескольких материалов. 6 \ text {V / m} \ right) \ left ( 1.{-3} \ text {m} \ right) \\\ text {} & = & 3000 \ text {V} \ end {array} \\ [/ latex]

Однако предел для расстояния 1,00 мм, заполненного тефлоном, составляет 60 000 В, поскольку диэлектрическая прочность тефлона составляет 60 × 10 6 В / м. Таким образом, тот же конденсатор, заполненный тефлоном, имеет большую емкость и может подвергаться гораздо большему напряжению. Используя емкость, которую мы рассчитали в приведенном выше примере для конденсатора с параллельными пластинами, заполненного воздухом, мы обнаружили, что конденсатор с тефлоновым заполнением может хранить максимальный заряд

[латекс] \ begin {array} {lll} Q & = & CV \\\ text {} & = & \ kappa {C} _ {\ text {air}} V \\\ text {} & = & (2.4 \ text {V}) \\\ text {} & = & 1.1 \ text {mC} \ end {array} \\ [/ latex]

Это в 42 раза больше заряда того же конденсатора, заполненного воздухом.

Диэлектрическая прочность

Максимальная напряженность электрического поля, при превышении которой изолирующий материал начинает разрушаться и становится проводником, называется его диэлектрической прочностью.

Микроскопически, как диэлектрик увеличивает емкость? За это отвечает поляризация изолятора. Чем легче он поляризуется, тем больше его диэлектрическая проницаемость κ .Вода, например, представляет собой полярную молекулу , потому что один конец молекулы имеет небольшой положительный заряд, а другой конец имеет небольшой отрицательный заряд. Полярность воды обуславливает ее относительно большую диэлектрическую проницаемость 80. Эффект поляризации лучше всего объясняется характеристиками кулоновской силы. На рис. 5 схематично показано разделение зарядов в молекулах диэлектрического материала, помещенных между заряженными пластинами конденсатора. Кулоновская сила между ближайшими концами молекул и зарядом на пластинах притягивает и очень сильна, поскольку они расположены очень близко друг к другу.Это притягивает больше заряда к пластинам, чем если бы пространство было пустым, а противоположные заряды находились на расстоянии d друг от друга.

Рис. 5. (a) Молекулы изоляционного материала между пластинами конденсатора поляризованы заряженными пластинами. Это создает слой противоположного заряда на поверхности диэлектрика, который притягивает больше заряда к пластине, увеличивая ее емкость. (б) Диэлектрик снижает напряженность электрического поля внутри конденсатора, что приводит к уменьшению напряжения между пластинами при одинаковом заряде.Конденсатор сохраняет тот же заряд при меньшем напряжении, что означает, что он имеет большую емкость из-за диэлектрика.

Другой способ понять, как диэлектрик увеличивает емкость, — это рассмотреть его влияние на электрическое поле внутри конденсатора. На рисунке 5 (b) показаны силовые линии электрического поля с установленным диэлектриком. Поскольку силовые линии заканчиваются зарядами в диэлектрике, их меньше, идущих от одной стороны конденсатора к другой. Таким образом, напряженность электрического поля меньше, чем если бы между пластинами был вакуум, даже если бы на пластинах был одинаковый заряд.Напряжение между пластинами составляет В, = Ед, , поэтому оно тоже уменьшается за счет диэлектрика. Таким образом, есть меньшее напряжение В, для того же заряда Q ; поскольку [латекс] C = \ frac {Q} {V} \\ [/ latex], емкость C больше.

Диэлектрическая проницаемость обычно определяется как [латекс] \ kappa = \ frac {E_0} {E} \\ [/ latex], или отношение электрического поля в вакууме к электрическому полю в диэлектрическом материале, и в конечном итоге связанных с поляризуемостью материала.

Большое и маленькое: субмикроскопическое происхождение поляризации

Поляризация — это разделение зарядов внутри атома или молекулы. Как уже отмечалось, планетарная модель атома описывает его как имеющее положительное ядро, вращающееся вокруг отрицательных электронов, аналогично планетам, вращающимся вокруг Солнца. Хотя эта модель не совсем точна, она очень полезна для объяснения широкого круга явлений и будет уточнена в других местах, например, в атомной физике. Субмикроскопическое происхождение поляризации можно смоделировать, как показано на рисунке 6.

Рис. 6. Художественное представление о поляризованном атоме. Орбиты электронов вокруг ядра слегка смещены внешними зарядами (показаны в преувеличении). Получающееся разделение зарядов внутри атома означает, что он поляризован. Обратите внимание, что непохожий заряд теперь ближе к внешним зарядам, вызывая поляризацию.

В атомной физике мы обнаружим, что орбиты электронов более правильно рассматривать как электронные облака с плотностью облака, связанной с вероятностью обнаружения электрона в этом месте (в отличие от определенных положений и путей движения планет на их орбитах). вокруг Солнца).Это облако сдвигается кулоновской силой, так что в среднем атом имеет разделенный заряд. Хотя атом остается нейтральным, теперь он может быть источником кулоновской силы, поскольку заряд, поднесенный к атому, будет ближе к одному типу заряда, чем к другому.

Некоторым молекулам, например молекулам воды, присуще разделение зарядов, поэтому они называются полярными молекулами. На рисунке 7 показано разделение зарядов в молекуле воды, которая имеет два атома водорода и один атом кислорода (H 2 O).Молекула воды несимметрична — атомы водорода отталкиваются в одну сторону, придавая молекуле форму бумеранга. Электроны в молекуле воды более сконцентрированы вокруг более заряженного ядра кислорода, чем вокруг ядер водорода. Это делает кислородный конец молекулы слегка отрицательным, а водородный конец — слегка положительным. Внутреннее разделение зарядов в полярных молекулах облегчает их выравнивание с внешними полями и зарядами. Следовательно, полярные молекулы обладают более сильными поляризационными эффектами и имеют более высокие диэлектрические проницаемости.Те, кто изучает химию, обнаружат, что полярная природа воды имеет множество эффектов. Например, молекулы воды собирают ионы гораздо эффективнее, потому что у них есть электрическое поле и разделение зарядов для притяжения зарядов обоих знаков. Кроме того, как было показано в предыдущей главе, полярная вода обеспечивает защиту или экранирование электрических полей в сильно заряженных молекулах, представляющих интерес в биологических системах.

Рис. 7. Художественная концепция молекулы воды. Существует внутреннее разделение зарядов, поэтому вода — полярная молекула.Электроны в молекуле притягиваются к ядру кислорода и оставляют избыток положительного заряда около двух ядер водорода. (Обратите внимание, что схема справа является приблизительной иллюстрацией распределения электронов в молекуле воды. На ней не показано фактическое количество протонов и электронов, участвующих в структуре.)

Исследования PhET: лаборатория конденсаторов

Узнайте, как работает конденсатор! Измените размер пластин и добавьте диэлектрик, чтобы увидеть влияние на емкость.Измените напряжение и посмотрите, как на пластинах накапливаются заряды. Наблюдайте за электрическим полем в конденсаторе. Измерьте напряжение и электрическое поле.

Щелкните, чтобы загрузить симуляцию. Запускать на Java.

Сводка раздела

  • Конденсатор — это устройство для хранения заряда.
  • Количество заряда Q , которое может хранить конденсатор, зависит от двух основных факторов — приложенного напряжения и физических характеристик конденсатора, таких как его размер.
  • Емкость C — это количество накопленного заряда на вольт, или [латекс] C = \ frac {Q} {V} \\ [/ latex].
  • Емкость конденсатора с параллельными пластинами составляет [латекс] C = {\ epsilon} _ {0} \ frac {A} {d} \\ [/ latex], когда пластины разделены воздухом или свободным пространством. [latex] {\ epsilon} _ {\ text {0}} [/ latex] называется диэлектрической проницаемостью свободного пространства.
  • Конденсатор с параллельными пластинами с диэлектриком между пластинами имеет емкость, определяемую выражением [латекс] C = \ kappa \ epsilon_ {0} \ frac {A} {d} \\ [/ latex], где κ — диэлектрик. константа материала.
  • Максимальная напряженность электрического поля, при превышении которой изолирующий материал начинает разрушаться и становится проводником, называется электрической прочностью.

Концептуальные вопросы

  1. Зависит ли емкость устройства от приложенного напряжения? А как насчет хранящегося в нем заряда?
  2. Используйте характеристики кулоновской силы, чтобы объяснить, почему емкость должна быть пропорциональна площади пластины конденсатора. Аналогичным образом объясните, почему емкость должна быть обратно пропорциональна расстоянию между пластинами.
  3. Объясните причину, по которой диэлектрический материал увеличивает емкость по сравнению с тем, что было бы с воздухом между пластинами конденсатора.Какова независимая причина того, что диэлектрический материал также позволяет приложить большее напряжение к конденсатору? (Таким образом, диэлектрик увеличивает C и допускает большее значение V .)
  4. Как полярный характер молекул воды помогает объяснить относительно большую диэлектрическую проницаемость воды? (См. Рисунок 7.)
  5. Искры возникают между пластинами заполненного воздухом конденсатора при более низком напряжении, когда воздух влажный, чем когда сухой. Объясните почему, учитывая полярный характер молекул воды.
  6. Вода имеет большую диэлектрическую проницаемость, но редко используется в конденсаторах. Объяснить, почему.
  7. Мембраны в живых клетках, включая клетки человека, характеризуются разделением заряда через мембрану. Таким образом, мембраны представляют собой заряженные конденсаторы, важные функции которых связаны с разностью потенциалов на мембране. Требуется ли энергия для разделения этих зарядов в живых мембранах, и если да, то является ли ее источником метаболизм пищевой энергии или каким-либо другим источником?

Рисунок 8.Полупроницаемая мембрана клетки имеет разную концентрацию ионов внутри и снаружи. Диффузия перемещает ионы K + (калий) и Cl (хлорид) в показанных направлениях, пока кулоновская сила не остановит дальнейший перенос. Это приводит к слою положительного заряда снаружи, слою отрицательного заряда внутри и, следовательно, к напряжению на клеточной мембране. Мембрана обычно непроницаема для Na + (ионы натрия).

Задачи и упражнения

  1. Какой заряд сохраняется в конденсаторе 180 мкФ, когда к нему приложено 120 В?
  2. Найдите накопленный заряд, когда 5.50 В подается на конденсатор емкостью 8,00 пФ.
  3. Какой заряд хранится в конденсаторе в Примере 1?
  4. Рассчитайте напряжение, приложенное к конденсатору 2,00 мкФ, когда он имеет заряд 3,10 мкКл.
  5. Какое напряжение необходимо приложить к конденсатору 8,00 нФ, чтобы накопить 0,160 мкКл заряда?
  6. Какая емкость необходима для хранения 3,00 мкКл заряда при напряжении 120 В?
  7. Какая емкость терминала большого генератора Ван де Граафа, учитывая, что он хранит 8?00 мкКл заряда при напряжении 12,0 МВ?
  8. Найдите емкость конденсатора с параллельными пластинами, площадь пластин которого составляет 5,00 м 2 , разделенных слоем тефлона 0,100 мм.
  9. (a) Какова емкость конденсатора с параллельными пластинами, площадь пластин которого составляет 1,50 м 2 , разделенных 0,0200 мм неопренового каучука? (b) Какой заряд он держит, когда к нему приложено 9,00 В?
  10. Интегрированные концепции. Шутник подает 450 В на 80.Конденсатор 0 мкФ, а затем бросает его ничего не подозревающей жертве. Палец пострадавшего обгорел от разряда конденсатора через 0,200 г мяса. Какое повышение температуры мяса? Разумно ли предполагать отсутствие изменения фазы?
  11. Необоснованные результаты. (a) Определенный конденсатор с параллельными пластинами имеет пластины площадью 4,00 м 2 , разделенные 0,0100 мм нейлона, и накапливает 0,170 Кл заряда. Какое приложенное напряжение? б) Что неразумного в этом результате? (c) Какие допущения являются ответственными или противоречивыми?

Глоссарий

конденсатор: устройство, накапливающее электрический заряд

Емкость: количество хранимого заряда на единицу вольт

диэлектрик: изоляционный материал

диэлектрическая прочность: максимальное электрическое поле, выше которого изоляционный материал начинает разрушаться и проводить

Конденсатор с параллельными пластинами: Две идентичные проводящие пластины, разделенные расстоянием

полярная молекула: молекула с внутренним разделением заряда

Избранные решения проблем и упражнения

1.

Добавить комментарий

Ваш адрес email не будет опубликован. Обязательные поля помечены *

*